image

Теория относительности завораживает своими парадоксами. Все мы знаем про близнецов, про возможности засунуть длинный самолёт в короткий ящик. Сегодня каждый выпускник школы знает ответы на эти классические загадки, а уж студенты-физики и подавно считают, что тайн в специальной теории относительности для них не осталось.

Всё бы хорошо, если бы не удручающе обстоятельство — невозможность сверхсветовых скоростей. Неужели никак нельзя быстрее?! — думала я в детстве. А может быть можно?! Поэтому приглашаю вас на сеанс, уж и не знаю, чёрной или белой магии имени Альберта Эйнштейна с разоблачением в конце. Впрочем для тех, кому покажется мало, я приготовила ещё и задачку.

UPD: Сутки спустя публикую решение. Много текста формул, графиков в конце.


К Альфе Центавра


Приглашаю вас занять места в нашем межзвёздном корабле, который направляется в сторону Альфы Центавра. От конечной точки маршрута нас отдаляют 4 световых года. Внимание, запускаем двигатели. Поехали! Для удобства пассажиров наш капитан установил такую тягу, чтобы мы ускорялись с величиной $a=g$ и ощущали привычную нам на Земле силу тяжести.

Вот мы уже прилично разогнались, пускай до половины скорости света $c/2$. Зададим казалось несложный вопрос: с какой же скоростью мы будем приближаться к Альфа Центавра в нашей собственной (корабельной) системе отсчёта. Казалось бы всё просто, если мы летим со скоростью $c/2$ в неподвижной системе отсчёта Земли и Альфы Центавра, то и с нашей точки зрения мы приближаемся к цели со скоростью $c/2$.

Тот, кто уже почувствовал подвох, совершенно прав. Ответ $c/2$ неверен! Тут надо сделать уточнение, под скоростью приближения к Альфа Центавра я называю изменение оставшегося расстояния до неё, делённое на промежуток времени, за который такое изменение произошло. Всё, разумеется, измеряется в нашей системе отсчёта, связанной с космическим кораблём.

Тут надо вспомнить, о лоренцевском сокращении длины. Ведь разогнавшись до половины скорости света мы обнаружим, что масштаб вдоль направления нашего движения сжался. Напомню формулу:

image

И теперь, если на скорости в половину скорости света мы измерим расстояние от Земли до Альфы Центавра, мы получил не 4 св. года, а всего лишь 3,46 св.года.

Получается, что только благодаря тому факту, что мы разогнались до $c/2$ мы уже уменьшили расстояние до конечной точки путешествия почти 0,54 св.года. А если мы будем не просто двигаться с большой скоростью, но ещё и ускоряться, то у масштабного фактора появится производная по времени, которая по сути тоже есть скорость приближения и плюсуется к $V$.

Таким образом помимо к нашей обычной, я бы сказала классической, скорости $V$ добавляется ещё один член — динамическое сокращение длины оставшегося пути, которое возникает тогда и только тогда, когда есть ненулевое ускорение. Ну что же, возьмём карандаш и посчитаем.

А тех, кому лень следить за вычислениями встречаю на другом берегу спойлера
$L$ — текущее расстояние до звезды по линейке капитана корабля, $t$ — время на часах в кают-компании, $V$ — скорость.

image

Уже здесь мы видим, что первая частная производная — это скорость, просто скорость $V$ со знаком минус, коль скоро мы приближаемся к Альфе Центавра. А вот второе слагаемое — тот самый подвох, о котором, подозреваю, не все задумывались.

Чтобы найти производную скорости по времени во втором слагаемом, надо быть аккуратным, т.к. мы находимся в подвижной системе отсчёта. Проще всего на пальцах её вычислить из формулы сложения релятивистских скоростей. Пусть в момент времени $0$ мы движемся со скоростью $V$, а через какой-то промежуток времени прирастили нашу скорость на $v$. Результирующая скорость по формуле теории относительности будет

image

Теперь соберём вместе (2) и (3), причём производную от (3) надо взять при $v=0$, т.к. мы рассматриваем малые приращения.

image

Полюбуемся на конечную формулу

image

Она удивительна! Если первый член — скорость — ограничен скоростью света, то второй член не ограничен ничем! Возьмите $L$ побольше и… второе слагаемое с лёгкостью может превысить $c$.

— Что-что! — не поверят некоторые.
— Да-да, именно так, — отвечу я. — Оно может быть больше скорости света, больше двух скоростей света, больше 10 скоростей света. Перефразируя Архимеда, могу сказать: «дайте мне подходящую $L$, и я обеспечу вам сколь угодно большую скорость.»

Что ж а давайте подставим числа, с числами всегда интереснее. Как мы помним, капитан установил ускорение $g $, а скорость уже достигла $V=c/2$. Тогда обнаружим, что при $L = 0.71$ светового года, наша скорость приближения сравняется со скоростью света. Если же мы подставим $L = 4$ световых года, то

image

Прописью: «три целых, три десятых скорости света».

Продолжаем удивляться


Давайте посмотрим ещё более внимательно на формулу (5). Ведь не обязательно садиться в релятивистский космический корабль. И скорость, и ускорение могут быть совсем маленькими. Всё дело в волшебной $L$. Вы только вдумайтесь!

Вот я села в машину и нажала на газ. У меня есть скорость и ускорение. И в этот самый момент я могу гарантировать, что где-то примерно сотне-другой миллионов световых лет впереди меня есть объекты, приближающиеся сейчас ко мне быстрее света. Для простоты я ещё не брала в расчёт скорость движения Земли по орбите вокруг Солнца, и Солнца вокруг центра Галактики. С их учётом объекты со сверхсветовой скоростью приближения окажутся уже совсем поблизости — не на космологических масштабах, а где-то на периферии нашей Галактики.

Получается, что невольно даже при минимальных ускорениях, например встав со стула, мы участвуем в сверхсветовом движении.

Удивляемся ещё


Посмотри на формулу (5) совсем-совсем пристально. Давайте узнаем не скорость приближения к Альфе Центавра, а наоборот скорость удаления от Земли. При достаточно большом $L$, например, на полпути к цели, мы можем обнаружить, что к нам приближается и Земля, и Альфа Центавра. Оправившись от удивления, конечно можно догадаться, что виной всему сокращение длины, которое работает не только вперёд, но и назад. Пространство за кормой космического корабля сжимается быстрее, чем мы улетаем от точки старта.

Несложно понять и другой удивительный эффект. Ведь стоит изменить направление ускорения, как второе слагаемое в (5) тут же поменяет знак. Т.е. скорость приближения может запросто стать нулевой, а то и отрицательной. Хотя обычная скоростью $V$ у нас по прежнему будет направлена к Альфе Центавра.

Разоблачение


Надеюсь, я вас достаточно сбила с толку. Как же так, нас учили, что скорость света максимальна! Нельзя приближаться к чему-либо быстрее скорости света! Но здесь стоит обратить внимание на присказку к любому релятивистскому закону. Она есть в любом учебнике, но кажется, что только загромождает формулировку, хотя именно в ней вся «соль». Эта присказка гласит, что постулаты специальной теории относительности работают «в инерциальной системе отсчёта».

В неинерциальной системе отсчёта Эйнштейн нам ничего не гарантирует. Такие дела!

Тоже самое, чуть более подробно и чуть более сложно
В формуле (5) содержится расстояние $L$. Когда оно равно нулю, т.е. когда мы пытаемся определить скорость локально относительно близких объектов, останется только первое слагаемое $V$, которое, разумеется, не превышает световую скорость. Никаких проблем. И лишь на больших расстояниях, т.е. не локально, мы можем получить сверхсветовые скорости.

Надо сказать, что вообще говоря, относительная скорость удалённых друг от друга объектов — понятие плохо определённое. Наше плоское пространство-время в ускоренной системе отсчёта выглядит искривлённым. Это знаменитый «лифт Эйнштейна» эквивалентный гравитационному полю. А сравнивать две векторные величины в искривлённом пространстве корректно, только когда они находятся в одной точке (в одном касательном пространстве из соответствующего векторного расслоения).

Кстати о нашем парадоксе сверхсветовой скорости можно рассуждать и по-другому, я бы сказала интегрально. Ведь релятивистское путешествия к Альфе Центавра займёт по собственным часам космонавта гораздо меньше 4 лет, поэтому поделив изначальное расстояние на затраченное собственное время, мы получим эффективную скорость больше скорости света. По сути это тот же парадокс близнецов. Кому удобно, может именно так и понимать сверхсветовое перемещение.

Вот и весь фокус. Ваша Капитанша Очевидность.

А напоследок я придумала вам домашнее задание или наброс для обсуждения в комментариях.

Задачка


Земляне и альфацентавры решили обменяться делегациями. С Земли стартовал космический корабль со скорость $c/2$. Одновременно с ним с Альфы Центавра навстречу отправилась летающая тарелка инопланетян с той же скоростью.

image

Каково расстояние между кораблями в системе отсчёта корабля землян в момент старта, когда они находились у Земли и Альфы Центавра соответственно? Напишите ответ в комментариях.

UPD: Решение


Итак решение задачи. Сначала рассмотрим её качественно.

Договоримся, что часы на Альфе, Земле, ракете и тарелке синхронизованы (это было сделано заранее), и старт по всем четырём часам состоялся в 12:00.

Рассмотрим пространство время графически в покоящихся координатах $(t, x)$. Земля находится в нуле, Альфа на расстоянии $L_0$ по оси $x$. Мировая линия Альфы Центавра, очевидно, просто идёт вертикально вверх. Мировая линия тарелки идёт с наклоном влево, т.к. она вылетела из точки $А$ в направлении Земли.

image

Теперь на этом графике пририсуем оси координат системы отсчёта ракеты, стартовавшей с Земли. Как известно, такое преобразование системы координат (СК) называется бустом. При этом оси наклоняются симметрично относительно диагональной линии, которая показывает световой луч.

image

Я думаю, в этот момент вам уже всё стало понятно. Смотрите, ось $x'$ пересекает мировые линии Альфы и летающей тарелки в разных точках. Что же произошло?

Удивительная вещь. Перед стартом с точки зрения ракеты и тарелка и Альфа находились в одной точке, а после набора скорости выясняется, что в движущеёся СК старт ракеты и тарелки не был одновременен. Тарелка, вдруг оказывается, стартовала раньше и успела немного приблизиться к нам. Поэтому сейчас в 12:00:01 по часам ракеты до тарелки уже ближе, чем до Альфы.

А если ракета разгонится ещё, она «перепрыгнет» в следующую СК, где тарелка ещё ближе. Причём такое приближение тарелки происходит только за счёт ускорения и динамического сжатия продольного масштаба (о чём собственно весь мой пост), а не продвижения ракеты в пространстве, т.к. ракета ещё по сути ничего и не успела пролететь. Это приближение тарелки, как раз и есть второй член в формуле (5).

Ну и кроме всего прочего надо учесть обычное лоренцевское сокращение расстояния. Сразу сообщу ответ, что при скоростях ракеты и тарелки по $c/2$ каждая расстояние

  • между ракетой и Альфой: 3,46 св. года (обычное лоренцевское сокращение)
  • между ракетой и тарелкой: 2,76 св. года

Кому интересно, давайте поколдуем с формулами в четырёхмерном пространстве
Такого рода задачи удобно решать с помощью четырёхмерных векторов. Бояться их не надо, всё делается при помощи самых обычных действий линейной алгебры. Тем более мы движемся только вдоль одной оси, поэтому от четырёх координат остаётся только две: $t$ и $x$.

Далее договоримся о простых обозначениях. Скорость света считаем равной единице. Мы, физики, всегда так делаем. :) Ещё обычно единицей считаем постоянную Планка и гравитационную постоянную. Сути это не меняет, зато чертовски облегчает писанину.

Итак повсеместно присутствующий «релятивистский корень» обозначим гамма-фактором для компактности записей, где $V$ — скорость земной ракеты:

image

Теперь запишем в компонентах вектор $\overrightarrow{OA}$:

image

Верхняя компонента — время, нижняя — пространственная координата. Корабли стартуют одновременно в неподвижной системе, поэтому верхняя составляющая вектора равна нулю.

Теперь найдём координаты точки $А$ в подвижной системе координат $(t', x')$, т.е. $(\overrightarrow{OA})'$. Для этого используем преобразование к движущейся системе отсчёта. Оно называется бустом и делается очень просто. Любой вектор надо умножить на матрицу буста

image

Умножаем:

image

Как мы видим, временная компонента этого вектора отрицательна. Это и значит, что точка $А$ с точки зрения движущеёся ракеты находится под осью $x'$, т.е. в прошлом (что и видно на рисунке выше).

Найдём вектор $\overrightarrow{AT}$ в неподвижной системе. Временная компонента — некоторый неизвестный пока промежуток времени $\Delta t_0$, пространственная — расстояние, на которое приближается тарелка за время $\Delta t_0$, двигаясь со скоростью $v$:

image

Теперь тот же самый вектор в системе $(t', x')$

image

Найдём обычную векторную сумму

image

Почему эту сумму я приравняла справа к таком вектору? По определению точка $T$ находится на оси $x'$, поэтому временная компонента $(\overrightarrow{OT})'$ должна быть равна нулю, а пространственная компонента — это и будет то самое искомое расстояние $L$ от ракеты до тарелки. Отсюда получаем систему двух простых уравнений — приравниваем временные компоненты отдельно, пространственные отдельно.

image

Из первого уравнения определяем неизвестный параметр $\Delta t_0$, подставляем его во второе уравнение и получаем $L$. Позвольте опустить простые вычисления и сразу записать

image

Подставив $V=1/2$, $v=1/2$, получаем

image

Комментарии (339)


  1. tmnhy
    30.09.2017 19:54

    Любопытно. Но нам надо прибыть в конечный пункт (остановиться в окрестностях Альфа Центавра), а не пролететь мимо него на всех парах. Т.е. в какой-то момент ускорение поменяет знак.

    Что произойдёт с масштабным фактором?


    1. Tyusha Автор
      30.09.2017 22:11

      Конечно разожмётся назад.


      1. BillFainder
        30.09.2017 22:20
        +1

        Торможение с такой скорости отнюдь не мгновенно, поэтому и расчёты будут совсем иные, но это не важно. Статья в общем-то о том, что для фотона, летящего к нам на скорости света с галактики в 2 млдрд. световых лет эти самые 2 млрд. лет проходят мгновенно, а пространство сворачивается в ноль. Вот только для сторонних наблюдателей это те же самые 2 млрд. лет и времени и расстояния, о чём уже сказали в комментариях


        1. Tyusha Автор
          30.09.2017 22:31
          +2

          Не стоит трогать фотоны, которые движутся ровно со световой скоростью, т.к. для них понятие собственного времени вообще не определено. Лучше говорить об ультрарелятивистских частицах. Но да, вы и комментаторы, конечно же, правы.


      1. MatiasGray
        02.10.2017 11:31

        Тогда в этом случае "расстояние" станет больше, и в сумме длины этапа ускорения и этапа торможения могут сравнятся с реальным расстоянием, для пилота?


        1. Tyusha Автор
          02.10.2017 12:42

          Нет, сумма всё равно будет меньше. Каждый пройденный ракетой в её системе отсчёта кусочек пути, будь то ускорение в начале или торможение в конце, всё равно короче того же фрагмента в неподвижной системе. Поэтому и сумма по-любому будет меньше 4 св. лет.


          1. MatiasGray
            02.10.2017 12:51
            +1

            Это замечательно.


  1. gasizdat
    30.09.2017 19:55

    Основное заблуждение в том, что в рассуждениях подразумеваются фантастические мгновенные измерения расстояний. Отсюда и сокращения расстояний с двух концов и прочие глупости.


    1. Tyusha Автор
      30.09.2017 22:19

      В чём именно стоит заблуждение? Да, масштабный фактор меняется. Если вы разгоняетесь мгновенно (оставим в стороне вопрос перегрузок), то мгновенно изменяются расстояния. И это не «подразумевается», а это так и есть, чёрт побери, нравится это или нет!

      Представьте, мы висим неподвижно между Землёй и Альфа Центавра, до каждой звезды по 2 световых года. Потом резко разогнались до c/2 к Альфе Центавра, и тут же расстояния мгновенно изменились. Теперь и до Земли и до Альфы по 1,73 световых года. Мгновенно, объективно! Никаких глупостей.


      1. Tyusha Автор
        30.09.2017 23:52
        +6

        Интересно, не это ли имели ввиду авторы поговорки: «Тише едешь — дальше будешь». Это доказывает, что древние знали кое-что от кого-то, кто прилетал сюда со скоростью c/2.

        Пора звонить на РЕН-ТВ!


      1. Googlist
        01.10.2017 00:31

        Вьі не можете разогнаться «мгновенно», за 0 времени, вьі можете какое-то время двигаться с огромньім ускорением.


        1. Tyusha Автор
          01.10.2017 01:19

          Это кинематика, а не динамика. Почувствуйте разницу.


        1. firuz1844
          02.10.2017 12:15
          +2

          У вас что, кнопка Ы не работает на клавиатуре? )


          1. sumanai
            02.10.2017 17:41
            +1

            Так часто пишут украинцы, у них такой кнопки походу нет, сужу только по их сообщениям.


            1. Stalker_RED
              03.10.2017 02:19
              +2

              На месте «ы» в украинской раскладке «и» с точкой. Как-то так wiki. Не смотря на то, что добавить раскладку в windows можно кликов за 5-10, многие почему-то пишут вот такими вот странными конструкциями.


      1. Hardcoin
        01.10.2017 03:38

        Нет, не объективно. Посчитать на листочке можно, но никакого способа измерить не вижу. Если представить висящие равномерно столбики-метки (или даже длинную линейку) — корабль будет полетать мимо метки "2 года", а не 1,73, независимо от того, какое у него ускорение. Разве нет?


        1. Hardcoin
          01.10.2017 03:47

          Впрочем, мы можем измерить время между двумя столбиками и оно нам покажет, что либо наша скорость не с/2, а выше (т.к. слишком быстро долетели до следующего), либо расстояние между столбиками стало меньше.


      1. aleksandros
        01.10.2017 14:19

        Да, быстрота измерения не причём. Это мысленный эксперимент, и если сейчас измерение занимает Х времени, через 50, 100 или 200 лет будет мгновенная технология. Так что от этого ничего не меняется.


  1. Welran
    30.09.2017 19:59
    +2

    Ммм, так вы и не стали двигаться быстрее скорости света. Просто расстояние до Альфы Центавра уменьшилось.


    1. Tyusha Автор
      30.09.2017 22:24

      Да, разумеется. Конечно присутствует некоторая путаница. Поэтому делаю специальную оговорку, что есть локальная скорость, а есть dL/dt. По сути это тоже скорость — быстрота преодоления расстояния.

      Вы едете на дачу, до неё 100 км. Прошла одна минута, до дачи осталось 99 км. Какая скорость? 60 км/ч. Так же строятся и рассуждения в статье.


  1. lostmsu
    30.09.2017 20:07
    -1

    На задачку ответ: 3,46 св.года (скопирован из статьи с ответом на аналогичную задачу с без тарелки).
    Но я всё же не понял, каким образом упоминание ОТО — разоблачение штуки с расстояниями? Ведь действительно лететь меньше по корабельному времени, даже с ОТО.


    1. Tyusha Автор
      30.09.2017 22:07

      Ответ неверный. Если бы так было всё просто, то я бы не стала задавать задачку.


    1. Kalobok
      30.09.2017 22:21
      +4

      ОТО здесь ни к чему. Все рассматривается в рамках СТО, просто надо учитывать ускорения. СТО вовсе не запрещает работать с ускорениями, как думают некоторые, просто основные формулы выведены для инерциальных систем отсчета, а при наличии ускорения они должны меняться.

      Что же касается задачи, думаю, подвох в относительности одновременности. С точки зрения земного корабля, момент его старта с земли не совпадает с моментом старта тарелки.


      1. Tyusha Автор
        30.09.2017 22:35

        Зачёт. :)

        Если будет интересно, думаю, разобрать эту задачку в качестве отдельной заметки.


        1. Kalobok
          30.09.2017 22:40
          +2

          Дело хорошее. :)


          1. Tyusha Автор
            30.09.2017 22:50

            Я поясню откуда взялась эта задача. Когда я для себя размышляла над тем эффектом «сверхсветовой скорости», о котором заметка, то никак не могла взять в толк, почему в вот втором члене (5) присутствует скорость?! Скорость ведь можно устранить. Если кто-то там, на Альфе не стоит и ждёт нас, а движется (то же там же на Альфе), какую скорость я должна подставлять во втором члене. Получалось, что моя скорость приближения зависит от того, двигается ли там кто-то на стороне мишени в конечной точке или нет.

            И удивительный ответ в том, что таки зависит! А задачка стала для меня своеобразной леммой. Сначала надо найти расстояние, а потом уже поделить на промежуток времени, размышляла я.


            1. Tyusha Автор
              01.10.2017 21:04
              +1

              Написала большой апдейт к посту с разбором решения задачи.


        1. Cast_iron
          01.10.2017 15:01

          Если наблюдатель находится на корабле Земли (на Земле) в момент старта, то с Альфа Центавра тарелка уже как 4 года должна лететь. Т.к. именно за это время до Земли дойдет световой/радио сигнал о старте тарелки с АЦ. Я правильно мыслю?


    1. Tyusha Автор
      01.10.2017 00:57
      +1

      Насчёт ОТО (общей теории относительности). Я хотела подчеркнуть, что парадокс происходит от нелокальности. Мы подсчитываем скорость до удалённого объекта. В СТО (специальной теории относительности), пока нет ускоренных систем отсчёта, проблемы тоже нет, можно рассчитывать скорость относительно сколь угодно далёких объектов.

      Ведь в ОТО никого не удивляет, что есть галактики удаляющиеся от нас со скоростью больше света. Это собственно те галактики, которые находятся примерно (у чёрту подробности) за космологическим горизонтом событий.

      Можно и здесь возмутиться. Но фишка в том (повторюсь), что не совсем корректно измерять относительную скорость далеко разнесённых объектов. Вернее измерять можно, но не надо возмущаться, что она больше скорости света, т.к. апеллировать к СТО в такой ситуации нельзя.

      Далёкая Альфа Центавра приближается быстрее света, но при этом относительная скорость метеоритов, мимо которых проносится космический корабль здесь и сейчас, конечно никак не превышает скорость света.

      В ускоренной системе отсчёта формально есть искривление пространства-времени, точно также, как в однородном гравитационном поле. А как я написала, в искривлённом пространстве скорость движения удалённого объекта может оказаться сверхсветовой. Однако в случае ускоряющегося лифта Эйнштейна искривление можно устранить соответствующим выбором так называемой сопутствующей системы координат.

      В то время же произвольное гравитационное поле нельзя накрыть такой системой координат, которая бы при этом покрывала всё пространство. Силу тяжести можно устранить только локально. Т.е. перейти в сопутствующую систему координат, с криками «А-а-а!» перерезав трос лифта на Земле.


  1. ShabanovYT
    30.09.2017 20:38
    +9

    Если скорость кк приближается к скорости света, происходит сокращение длины и времени, по корабельным часам. При достаточно большой скорости можно долететь до альфа Центавра за несколько часов. К сожалению, для внешнего наблюдателя все будет выглядеть несколько иначе.


    1. Ilya81
      02.10.2017 11:12

      Вот я о том ж подумал — для пассажиров комического корабля полёт может занять и час, и минуту, но вот если вернуться на землю, там пройдёт уже 10 лет, и от этого никуда не деться. А если куда-то дальше — и время на нашей планете пройдёт соответствующее за время полёта.


  1. tolkach88
    30.09.2017 20:45
    +4

    фигня это все — нам важно туда прилететь и вернуться в реальном времени — то что что-то там на пол метра мнимо укоротиться из-за замедления времени на корабле и так всем понятно. экипаж сел прилетел туда за 3 дня вернулся за столько же, а на земле уже жизни нет — потому что прошло миллион лет))) нас такой расклад не устроит)


    1. Sadler
      30.09.2017 21:32

      За исключением случая, когда это «корабль-ковчег», не имеющий целью возвращение.


    1. Dreablin
      30.09.2017 22:05
      +2

      А зачем возвращаться?
      Очевидно что такие полеты должны быть в 1 сторону.Нашли планету в 10 000 св лет пригодную для жизни, отправили корабль, они долетели за 3-5 леи и живут там. В связи между планетами нет никакого смысла — сигнал 10 тыс лет идет.
      Просто колонизация независимыми «семенами жизни».


      1. BillFainder
        30.09.2017 22:22
        +1

        С расстояния в 10 000 св. лет непросто понять, подходит ли планета, так что хоть раз послать робота туда-сюда придётся


        1. artemev
          01.10.2017 00:04

          В этом мало смысла. За такое время все может очень сильно поменяться. Ну там метеорит шарахнет или еще какой катаклизм случится.


          1. x86d0cent
            01.10.2017 01:12
            +2

            Вероятность того, что за 10 (да и пусть даже 50) тысяч лет условия на планете изменятся кардинально, очень мала. По геологическим меркам это очень небольшой период времени. Если на планете есть вода и кислород — вряд ли они за это время исчезнут. Ну а то, что может на несколько градусов похолодать — это уже не столь критично.


            1. vedenin1980
              01.10.2017 01:38

              По геологическим меркам — возможно, зато по биологическим уйма времени: могут мутировать вирусы, появится особо опасные виды животных или растений, местные «обезьяны» могут построить развитую цивилизацию (много ли было признаков разумной деятельности человека 50 тыс. лет назад?) или прилетят инопланетяне.

              Далеко не факт, что прилетевшая экспедиция сможет справиться с изменившимся обстоятельствами (и это при условии, что разведка не пропустила чего-то важного вроде разумной жизни). Если билет в один конец, то довольно рискованное операция получается, можно вспомнить сколько критики было о Маркс One, а тут точно помощи не дождешься. А если есть возможность вернуться обратно — неизвестно что вообще на Земле будет к тому моменту.


              1. Dreablin
                01.10.2017 02:11
                +2

                Ну, надо будет жить среди местных.
                Похимичить с атомами воды, превратив в вино, глушить рыбу табельной гранатой пилота… И можно неплохо устроиться, если местные не успели особо развиться :)


              1. Hardcoin
                01.10.2017 03:55

                То, что там разовьётся за это время разумная жизнь, вы рассматриваете, как вероятное событие, а то, что мы прилетим, а там бесполезный для жизни шар — как невероятное? Все же проверить заранее сотню планет сотней зондов разумнее. И лететь к самым лучшим из этой сотни.


                А ещё лучше двигатель Альбукерке построить. :)


                1. BaLaMuTt
                  02.10.2017 10:14
                  +1

                  двигатель Альбукерке
                  Алькубьерре)))


              1. x86d0cent
                01.10.2017 10:24
                +1

                Все, о чем вы написали, относится только к обитаемым планетам. Нет жизни — нет всех этих проблем :)
                Но даже по биологическим меркам это немного — появление за такой период времени нового, кардинально другого, а потому «опасного вида животных или растений» крайне маловероятно. Развитие высокоразвитой цивилизации из «обезьян» — тоже вряд ли (по крайней мере у нас 50000 лет назад уже начинался верхний палеолит).
                То, что разведка может что-то пропустить, это уже совсем другой вопрос. А относительно наличия рисков в принципе вроде бы никто и не спорит.


      1. ClearAirTurbulence
        01.10.2017 09:36

        Вот и американцы сначала так думали, но уже давно забыли про изоляционизм.


      1. BaLaMuTt
        02.10.2017 08:27
        -1

        Думаю что если отправить корабль со скоростью света в сторону планеты за 10000 св.лет то колонистов по прилёту будут встречать потомки других колонистов вылетевших позже на пару тысяч лет уже со сверхсветовой скоростью, да и связь между планетами уже будет налажена без пингов в 10000 лет.


  1. ruslan_zasukhin
    30.09.2017 20:56
    -22

    А вот когда я был в школе еще, для меня было загадкой — ну как кто то мог додуматься что тела должны сокращаться при движении. С какого перепугу? Позже я узнал (более глубоко) что на то время все исходили из теории эфира. И раз есть эфир и тело движется в нем — то ессно испытывает сопротивление и сокращение согласно такой то формуле.
    Кстати говоря аналогичную непонятку имел читая принцип Гуюйгенса. Опять же, если знать про эфир то этот принцип становится очевидным.
    Даже Е = мс2 оказывается была выведена еще ДО Эйнштейна, в 1897 году Томпсоном. Просто как сумма кинетической энергии всех частиц эфира магнитного и электрического полей:
    E = E1 + E2 = mc2/2 + mc2/2 = mc2

    Красиво просто элегантно.

    Теперь, когда СТО взяла формулу mc2 и лоренцевы сокращения, спрашивается и что мы подтверждаем? Теорию эфира? ))

    P.S. да да я в курсе про «все хорошо без эфира». Конечно хорошо пока вы пользуетесь формулами уже выведенными с его помощью ))

    P.S.2 А вы знаете что Морли утверждал что он нашел эфирный ветер? Причем не в 1903 году, а в 1925, 1929 и даже 1936-1940. Прямой линк для желающих убедиться. Отчеты Миллера
    ether-wind.narod.ru/Miller_1926_PZM
    ether-wind.narod.ru/Miller_1934

    А вам/нам говорят что эксперимент был не удачен, эфирный ветер не обнаружен. Интересно кто врет и почему?


    1. Arcpool
      30.09.2017 21:06
      +5

      Все смешалось в доме Облонских. СТО, КМ, Эйнштейн, Морли.
      Вы бы формулу для начала расписали, что у вас там Е1 и Е2 значат.


      1. ruslan_zasukhin
        30.09.2017 21:30
        -3

        не у меня а у Томпсона. Читайте внимательней. Я написал — Е1 энергия магнитного поля, E2 — электрического поля. Мы ж говорим про электро-магнитного поля да? Его изображают кольца горизонтальные — вертикальные.


        1. Arcpool
          30.09.2017 21:51
          +3

          Вы писали про сумарную кин. энергию каких-то частиц, и зачем-то приплели поля потом. Вот я и попросил конкретизировать. И не у Томпсона, а таки у Вас, пока ссылку не приведете.


          1. ruslan_zasukhin
            30.09.2017 21:55
            -4

            уже привел ниже. Я не ленивый еще раз. Докажите себе что вы тоже не ленивый и поищите потом самостоятельно более глубоко :)

            www.e-reading.club/chapter.php/1028159/68/100_znamenityh_uchenyh.html

            В своих работах «Электричество и материя», «Материя и эфир», «Структура света», «Фарадеевы силовые трубки и уравнения Максвелла» Томсон последовательно развивал вихревую теорию материи и взаимодействий.
            Известная работа ученого «Трактат о движении вихревых колец» была удостоена приза Адамса в 1884 году. Исходя из вихревой теории эфира, Томсон вывел формулу Е= mc 2 задолго до Эйнштейна.


            1. Tyusha Автор
              30.09.2017 23:03
              +4

              Вы приводите правильные факты, но не совсем правильно их трактуете. И E=mc2 и преобразования Лоренца были выведены до Эйнштейна. Всё это уже содержится в уравнениях Максвелла 1863 (или какого-то там, не помню точно) года.

              Вот только до Эйнштейна все эти формулы были занятным казусом. Проблема была в том, что электромагнетизм выглядел внутренне противоречивым. И только Альбер Германович набрался смелости сказать: это не просто формулы, так оно и есть на самом деле.

              Стоит отметить, что Эйнштейн для СТО — просто знаковая фигура. В «узких кругах ограниченных лиц» всё понятно было и до него, но все не решались это произнести вслух.


              1. Tyusha Автор
                30.09.2017 23:11
                +5

                Под все эти сокращения длины, времени и mc2 просто пытались подоткнуть эфирные костыли. И да у Томпсона это получилось. Но факт также в том, что «костыли» в чём-то другом противоречили эксперименту. Ничего страшного в этом нет, так развивается наука. Томас Кун, научные революции, всё вот это. И именно поэтому Эйнштейн — фигура первой величины, а Томпсон — что ни говори, второй.


    1. a2v
      30.09.2017 21:21
      +2

      Откуда известно, что Е1 и Е2, чем бы они ни были, равняются мс2/2?


      1. ruslan_zasukhin
        30.09.2017 21:35
        -2

        На то время мыслили эфиром. Электро магнитное поле — было некое движение эфира. Следовательно кинетическая энергия. Берем все частицы эфира в такой то области (которая нас интересует) это будет м. Частицы эфира участвующие в электро-магнитных полях предположили видимо — дижутся со скоростью с. Так что это простая формула кинетической энергии всех частиц.


        1. sumanai
          30.09.2017 21:42
          +1

          На то время мыслили эфиром.

          А вы не путаете середину XVIII века и начало XX?


          1. ruslan_zasukhin
            30.09.2017 21:44
            -1

            Нет я не путаю. А вы как оказываеся даже и не знали. Может прочитать здесь например для начала, а потом поискать его работы

            www.e-reading.club/chapter.php/1028159/68/100_znamenityh_uchenyh.html

            В своих работах «Электричество и материя», «Материя и эфир», «Структура света», «Фарадеевы силовые трубки и уравнения Максвелла» Томсон последовательно развивал вихревую теорию материи и взаимодействий.
            Известная работа ученого «Трактат о движении вихревых колец» была удостоена приза Адамса в 1884 году. Исходя из вихревой теории эфира, Томсон вывел формулу Е= mc 2 задолго до Эйнштейна.


            1. sumanai
              30.09.2017 22:05
              +4

              Может прочитать здесь например для начала, а потом поискать его работы

              Один физик против всего научного сообщества?
              Исходя из вихревой теории эфира, Томсон вывел формулу Е= mc 2 задолго до Эйнштейна.

              Но это не значит, что теория эфира правильная. Про Е= mc 2:
              Как и любой велосипед, была кем только ни открыта: Оливером Хевисайдом (1889; ЧСХ, он получил великую формулу как побочный результат сведения системы 22 уравнений Максвелла в векторную форму, которую изучает каждый студент, но, снова СХ, посчитал её дефектом математики, а не окружающего мира), Толвером Престоном (1895), Кельвином (1903), Олинто де-Претто (1903), Анри Пуанкаре (1904), Фритцем Хазенорлом (1904, ошибся коэффициентом), Максом Планком (1907). Но мы ведь не ищем простых путей.

              © Луркоморье


          1. ruslan_zasukhin
            30.09.2017 21:49
            -2

            Кроме того выше я дал прямые ссылки на отчеты Миллера 1905 — 1940 годы. На всякий случай скажу это XX век. Почитайте — удивитесь.
            Еще раз это тот самый человек, который ЯКОБЫ не нашел эфирный ветер. Но он лично пишет совсем другие резульаты. Опс?
            Но зачем читать? В учебниках то не так все написано :)


            1. sumanai
              30.09.2017 22:07
              +2

              Но он лично пишет совсем другие резульаты.

              Очевидно, что лично он не хотел признавать поражение. Поэтому в науке признаются результаты, которые смогли повторить другие, независимые группы.


            1. d1mk0
              01.10.2017 00:08
              +1

              Это же надо перелопатить столько литературы, чтобы троллить обитателей хабра. Наш мир вообще компьютерная симуляция.


              1. Hardcoin
                01.10.2017 04:01

                Он не лопатил. NPC заскриптованы, в них модель рассуждений вшита заранее.


    1. MonstraG
      30.09.2017 22:05
      -1

      Но и до и после Морли люди повторяли эти эксперименты и не получили таких же результатов.


      1. Tyusha Автор
        01.10.2017 00:03
        +5

        «О чём на самом деле свидетельствуют результаты наблюдений так называемых „гравитационных волн“ на установках LIGO — более совершенных приборов, чем тот, чтобы был у Морли? О чём вот уже более 100 лет предпочитают молчать учёные? Почему научное сообщество скрывает существование эфира? Смотрите на РЕН-ТВ после рекламы».


        1. Dessloch
          02.10.2017 11:53
          -1

          Демагогический приём «захихикивания» проблемы или вопроса всем знаком. Не утруждайтесь зря. Скажите ваше личное мнение-как же распространяется гравитация?


    1. Hardcoin
      01.10.2017 03:57
      +3

      Хреновый у вас был учитель физики, сочувствую.


  1. Vespertilio
    30.09.2017 21:19

    "… скорость приближения и плюсуется к ." чему?
    "… промежуток времени прирастили нашу скорость на ." сколько?

    Почему некоторые предложения на полуслове обрываются?


    1. boris768
      30.09.2017 22:31

      мне кажется, что у вас формулы плохо прогружаются


      1. Tyusha Автор
        01.10.2017 01:24
        +1

        Скорее всего Vespertilio читает со смартфона, там всегда фигня с формулами.


  1. Aytuar
    30.09.2017 22:05
    +2

    Ну для элементарных частиц этот эффект особенно заметен, например для кванта света, он двигается со скоростью света, и у него часы остановлены, он живёт бесконечно долго пока не столкнётся с другой частицей. А для частиц которые движуться со скоростью близкой к скорости света время течёт замедленно, и поэтому их время жизни (распада) увеличиватся за счёт этого.


  1. Pelemeshka
    30.09.2017 23:15

    К сожалению вы не учитываете что при скоростях сравнимых со скоростью света изменяются не только размеры (Лоренцево сокращение), но также восприятие времени (релятивистское замедление времени) и масса объекта (релятивистская динамика).
    Ответ с/2 на самый первый вопрос верен т.к. для наблюдателя на корабле течение времени и расстояние изменяется пропорционально (если посмотреть на формулы коэффициент изменения от скорости одинаковый). Давайте разберем ваш пример — корабль летит от Земли к Алфе Центавра (4 св.года) со скоростью в половину скорости света. Для наблюдателя на земле он пройдет это расстояния за 8 лет, но для наблюдателя на корабле он пройдет расстояние в 3,46 св.года за 6,92 лет, и если поделить расстояние на время опять получим скорость в половину скорости света.
    Второй момент — если взглянуть на движение с постоянно приложенной силой (с ускорением) учитывая все 3 изменения одновременно, то возникают интересные особенности, а именно:
    1. Из-за увеличивающиеся массы (см формулы про массу в покое и движении) приложенная к объекту сила (работа двигателя) будет иметь все меньший эффект, т.е. ускорение будет уменьшатся. Если вначале ускорение будет 1g, то при скорости в с/2 ускорение будет равняться 0,86g, и далее по мере приближения к скорости света ускорение будет уменьшаться соответственно.
    2. Что бы сохранить ускорение придется тратить все больше энергии, при приближении к скорости света количество необходимой энергии приближается к бесконечности.
    3. Из-за изменения восприятия времени наблюдатель на корабле при приближении к скорости света за несколько минут увидит как мимо проносятся галактики и целые местные группы, как возникают новые и умирают старые миры. Кто знает куда он улетит за эти минуты, может он упрется в край вселенной (шутка), а, тем временем, для наблюдателя на земле проходят миллиарды лет.


    1. Tyusha Автор
      30.09.2017 23:21

      «Восприятие времени» в формулах учитывается, т.к. там дифференцируется по собственному времени корабля.

      В нижней части комментария вы всё правильно написали про особенности. Но хочу прокомментировать:

      1. Масса сейчас не причём, это динамика, а в статье кинематика. Понятно, что с точки зрения неподвижной системы ускорение будет снижаться, сколько ни газуй на корабле, да так, что скорость света не превысится. Но описанный в статье эффект работает даже если вы ненадолго включили ускорение.

      2. Мне категорически не нравится фраза «восприятие времени». Это не восприятие, а самое настоящее, разобъективнейшее, течение времени. Понимаю, что вы это понимаете, но давайте будем точны, т.к. нас читаю менее подготовленные люди.


    1. Tyusha Автор
      30.09.2017 23:30

      Давайте разберем ваш пример — корабль летит от Земли к Алфе Центавра (4 св.года) со скоростью в половину скорости света. Для наблюдателя на земле он пройдет это расстояния за 8 лет, но для наблюдателя на корабле он пройдет расстояние в 3,46 св.года за 6,92 лет, и если поделить расстояние на время опять получим скорость в половину скорости света.


      Да, всё так, но только при отсутствии ускорения. Поставьте в формуле (5) a=0, то и получите.


    1. Hardcoin
      01.10.2017 04:11

      В системе отчёта, движущейся с/2 относительно земли (как корабль, но без ускорения), ускорение будет все же 1g, а количество энергии, нужной для продолжения ускорения, не возрастёт. Потому что корабль в этой системе движется еле-еле.


  1. Serge3leo
    30.09.2017 23:56

    Длину в одной системе отсчёта не стоит делить на время в другой системе отсчёта, вот и разоблачение Вашего 2+2=5.

    А для преобразований Лоренца инерциальность без надобности.


    1. Tyusha Автор
      01.10.2017 00:19

      Вычисления происходят именно что в одной системе координат — системе корабля. Вот как раз, если делать, как вы говорите, сверхсвета не получится.


  1. Hemeg
    01.10.2017 00:08
    +9

    А просто отправить автора учить СТО нельзя? Я понимаю, что надо аргументировать, но чудовищно лень.
    СТО — теория о наблюдении. Если кто то движется относительно чего то, то он видит, что то. В Любой системе координат остается постоянным только пространственно-временной интервал image, вот это ds^2 из какой инерциальной системы не измеряй будет одним.
    Перестаньте оперировать СТО как ньютовской физикой, это там есть скорость, время и расстояние, в СТО есть скорость относительно чего то, время с точки зрения кого то, расстояние измеренное кем то движущимся с отностительной скоростью.
    Если вы летите к Альфе Центавре со скоростью относительно Земли V1, то Земляне увидят, что вы долетели до цели через время T1 и все, и только. Возьмите свой пост и перепишите его в рамках терминов принятых в СТО, и проблемы рожденные игрой ума уберутся.


    1. Tyusha Автор
      01.10.2017 00:13

      А просто отправить автора учить СТО нельзя?

      Боюсь признаться, что я уже давно отправилась… учить СТО… э-э-э… студентов МФТИ. :) Хотя по-правде говоря, в основном преподаю кванты.


      1. Hemeg
        01.10.2017 00:32

        А зачем такие наивные ошибки? Это неуважение к местной публике, или попытка оценить общий уровень, читателей?


        1. Tyusha Автор
          01.10.2017 01:08
          +1

          Пожалуйста конкретно, где ошибки? Если вы хотите измерять скорость относительно Земли, пожалуйста, измеряйте, а я хочу провести расчёты в неинерциальной системе отсчёта, и в этом суть поста! Вы пытаетесь решать другую задачу и получаете другой ответ. Конечно он не совпадает с ответом на мою задачу.

          Хочу не просто пробубнить: «в ускоренной системе отсчёта нельзя применять постулаты теории относительности», а проиллюстрировать это наглядно и эффектно, как именно эти постулаты там нарушаются с формулами и цифрами.


          1. Hemeg
            01.10.2017 01:58
            +2

            Неинерциальная здесь не решается, нужно ОТО.

            Каково расстояние между кораблями в системе отсчёта корабля землян в момент старта, когда они находились у Земли и Альфы Центавра соответственно?


            Так нельзя ставить задачу в СТО, нет такого момента.


            1. Hemeg
              01.10.2017 02:02
              +3

              Нет в СТО понятия одновременности.


              1. Tyusha Автор
                01.10.2017 14:50
                +3

                Понятие есть, просто оно зависит от системы отсчёта.


              1. aleksandros
                01.10.2017 15:27
                -1

                Мысленный эксперимент. Из сотен миллиардов-триллионов планет учёные нашли две такие, расстояние между которыми не изменяется. Ровно посередине между ними сделали командный пункт к которому от планет подвели провода. В пункте учёный нажимает на кнопку, по проводам идёт ток и на каждой планете загорается лампочка. Обе лампочки зажгутся одновременно, разве нет?


                1. Hardcoin
                  01.10.2017 15:36
                  +1

                  А как узнать? Вам в мысленном эксперименте нужно придумать дальнодействие, с помощью которого узнать, загорелись или ещё нет.


                  Эдакий глобальный наблюдатель, который за обоими лампочками сразу смотрит. А его нет и он невозможен.


                  1. aleksandros
                    01.10.2017 20:57
                    -1

                    Да господи, элементарно. На планетах сидит по ещё одному учёному, которые увидев загоревшуюся лампочку нажимают кнопку обратной связи. После чего сигнал идёт в командный пункт по другим проводам к лампочкам А и Б соответственно. Там «главный» учёный смотрит одновременно ли они зажглись и делает вывод.


                    1. vedenin1980
                      01.10.2017 21:08

                      Там «главный» учёный смотрит одновременно ли они зажглись и делает вывод.

                      Он не сможет доказать что они зажглись одновременно, а только то что сигнал от командного пункта до лапочек А и Б и обратно приходит одновременно.

                      Представим что на самом деле от командного пункта до лапочки А сигнал идет на 1 секунду больше чем обратно. А до лампочки B одинаково, тогда обратно сигнал придет одновременно, но лампочки загорятся в разное время.


                      1. aleksandros
                        02.10.2017 09:26
                        -1

                        Ну мы же умные, мы строго проследили, чтобы длина проводов была одинакова. А значит причин для задержек нет.


                        1. vedenin1980
                          02.10.2017 10:33
                          -1

                          1. Кто вам сказал, что если длина проводов одинакова то причин для задержек нет? Может рядом с одним из проводов пролетала ЧД, которая замедляет прохождение сигнала? Вообще кто вам обещал что сигнал в проводах равной длины всегда идет одновременно? ОТО? Во-первых это не так, во-вторых, вы как раз относительность времени и пытаетесь в мысленном эксперименте опровергнуть.

                          2. Если вы забыли, вы в вашем мысленном эксперименте доказываете одновременность прихода сигнала в точки А и В. А сейчас ваше доказательство свелось «ну длина же проводов одинакова, значит сигнал придет одновременно». Это получается как в анекдоте «мамой клянусь теорема верна»


                          1. aleksandros
                            02.10.2017 10:46
                            +1

                            Вы извините, но мне ваши аргументы напоминают «ракеты не могут лететь со скоростью близкой к скорости света, так как столько кочегаров в них не поместится». Расскажите, почему сигналы в проводах равной длины приходят не одновременно.
                            Не нравятся провода, заменим их лазерами. Дополнительно добавим то, что учёные предварительно удостоверились, что поблизости нет чёрных дыр.


                            1. vedenin1980
                              02.10.2017 11:13
                              -1

                              Расскажите, почему сигналы в проводах равной длины приходят не одновременно.Не нравятся провода, заменим их лазерами.

                              Потому что пространство в ОТО не однородно, вращающаяся Черная дыра может расширять пространство, Черная дыра в окрестностях может его сжимать, гравитационные волны тоже расширяют/сжимают пространство/время. Кстати, Черная дыра это только наглядный пример, любая масса влияет на искривление пространства.

                              Дополнительно добавим то, что учёные предварительно удостоверились, что поблизости нет чёрных дыр.

                              Черная дыра это только пример, надо тогда убрать вообще любые крупные массы (например, звезды) с место прохождения лазера/проводов. Тогда эксперимент теряет смысл. Вы же доказываете что утверждение «Нет в СТО понятия одновременности» ложно. А если вы начинаете придумывать какие-то граничные условия, вроде «пространство абсолютно пустое и не искаженное», ваш мысленный эксперимент докажет что в некоторых случаях одновременность бывает (но это мы и так знаем, то что лампочки в комнате загорятся одновременно и так понятно).

                              P.S. Не пытайтесь хитрыми мысленными экспериментами опровергнуть СТО или ОТО, такие эксперименты придумывали уже сотню лет и пока опровергнуть никому не удалось.


                1. ariksu
                  01.10.2017 18:18
                  +2

                  А теперь внимание, правильный ответ: зависит от наблюдателя. (классическая книжка про СТО/ОТО ещё советская «бегство от удивлений»)
                  Рассмотрим трёх наблюдателей, все три находятся в точке включения посередине, но один неподвижен, другой двигается от А к Б, третий от Б к А. Так вот, неподвижный осознает что лампочки зажглись одновременно. Подвижный в сторону Б, увидит что лампочка Б загорелась раньше. Почему? Да расстояние до неё меньше, току по проводам меньше до неё бежать. Подвижный в сторону А увидит что А загорелась раньше. По той же самой причине.
                  Одновременности нет.


                  1. Iron_69
                    01.10.2017 21:02

                    ИМХО Вы ошибаетесь, если в в тот момент когда фронты световых волн достигнут командного пункта (середины) и в этот момент все три наблюдателя окажутся в одной точке, в независимости от скорости и направления — они увидят что обе лампочки зажглись одновременно. Просто для движущихся наблюдателей спектр излучения лампочек будет различен. Иначе нарушается принцип постоянства скорости света, в независимости от скорости источника и скорости приемника.


                  1. michael_vostrikov
                    01.10.2017 22:17
                    +1

                    Мне кажется, вы путаете одновременность событий и скорость распространения сигнала. На скорость распространения можно сделать поправки и вычислить истинное время события.


          1. itlen
            01.10.2017 11:49

            Это не ошибки. А осознанное заигрываение. Пост по итогу выглядит примерно:
            1. СТО постулирует невозможность сверхсветовых скоростей.
            2. Однако смотрите: финт ушами!
            3.… Вот и опровергли!
            4. Но не в рамках СТО. Такие дела.

            Пост не рассчитан на тех кто разбирается, но тогда, имхо, не хватает развернутого вывода что, вот этим финтом (рассмотрение в неинерциальных системах) объясняются вообще все парадоксы СТО, и поэтому когда кто-то будет приводить их (парадоксы) в качество доказательства — помните про систему отсчета!

            Но продолжайте :)


            1. Tyusha Автор
              01.10.2017 12:12

              Спасибо, что почти поняли. А то многие твердят, что это не так, вот посмотрите в системе Земли или сисиеме корабля без ускорения, там нет сверхсветовой скорости.

              Да, там нет. Но я-то решаю другую задачу, задачу в неинерциальной системе отсчёта! Вопреки вашему пункту 4 это делать можно делать в СТО, почему нет?! И там честно получается сверхсвет.

              То, что это финт— не согласна. Найти скорость в неинерциально СО — нормальная задача, просто высвечивающая непривычные моменты СТО, и учить быть незашоренным под влиянием постулатов, которые большинство применяют не внимательно.

              В результате таких упражнений можно только лучше и глубже понять СТО, ещё раз посидеть и подумать над ней. Когда её изучают не грузят ускоренными система отсчёта, и так парадоксов хватает.


              1. itlen
                01.10.2017 12:31
                +3

                Собственно я об этом же, нужен развернутый вывод что это просто математика, результаты расчетов которой не имеют физического смысла в нашей вселенной, что подобным образом (вводя различные допущения) можно вообще в любой теории найти парадоксы и нестыковки. Что это просто разминка для мозгов, которая помогает лучше понять СТО.


                1. drauren
                  02.10.2017 01:49

                  Значит ли это, что если я нахожусь на этом самом корабле, движущимся со скоростью c/2, и хочу пролететь эти самые 4 световых года, то по моим внутренним биологическим часам я постарею на все 8 лет? Или же для меня поездка займет чуть меньше времени, т.к. для моей собственной системы отсчета расстояние до конечной точки будет короче?

                  Прошу прощения, если я глупость какую написал, я просто пытаюсь разобраться.


                  1. Tyusha Автор
                    02.10.2017 01:53
                    +2

                    Если это 4 года в системе отсчёта Земли, то разогнавшись до c/2 вы тут же обнаружите, что путь до Альфы сжался до 3,46 св.года. Теперь если вы их пролетите с этой скорость с/2, то потратите на это по собственным часам (хоть механическим, хоть биологическим, хоть каким) 3,46*2 = 6,92 года. В то же время по часам неподвижного наблюдателя мы долетите за 8 лет.


                  1. itlen
                    02.10.2017 13:14
                    -1

                    Движение — жизнь. Есть такая поговорка.

                    Тот кто движется в системе отсчета в пространстве, движется медленее во времени в ней же, соответственно живет дольше в ней же. В своей неинерциальной системе отсчета Корабль вы не движетесь в пространстве, только во времени, в ней вы живете с обычной скоростью секунда в секунду. Пространство-время это одна система координат. Нельзя рассматривать движение в пространстве без изменения скорости движения движения во времени.

                    Скорость в пространстве «отбирает» у скорости движения во времени. Представьте машину на гонках. Была скорость 0 км/ч в пространстве и 1 s/s (секунда в секунду скорость во времени), машина начала движение, скорость стала 100 км/ч и скорость времени стала 0,5 s/s.

                    Но ваша скорость 0,5 s/s во времени стала для того, для кого ваша скорость в пространстве изменилась с 0 до 100 км/ч.

                    Для системы отсчета Машина ваша скорость осталась 0, вы как сидели в ней неподвижно на старте, так и остались в ней же неподвижно когда машина стала двигаться с 100 км/ч, соответственно и время осталось со скоростью 1 s/s. А для людей на трибунах ваша скорость изменилась. Скорость в пространстве и скорость во времени.


    1. Hardcoin
      01.10.2017 04:30
      +2

      Можно вопрос? Не уверен, что точно формулирую, поправьте. Если t, это координата и если у второго близнеца (извините, что опять про них) прошло меньше времени, то как они встретились? По координате x он улетел, потом вернулся (переместился, выходит, на ноль). А по t они переместились по-разному. Но тогда они должны быть в разных точках, когда второй вернулся обратно, разве нет?


      Или t, это все же не координата, а некая условность?


      1. zuborg
        02.10.2017 15:09

        t это координата, но не одна, а в системе отсчета каждого из близнецов. t1 и t2, грубо говоря.
        Исходно t1 = t2. А после полета они расходятся: t1 = t2 — delta. Там, где для первого близнеца событие происходит в момент 50лет 0дней 0секунд — для второго оно же отмечается по координате 50лет 10дней 0секунд.


        1. Hardcoin
          02.10.2017 15:34

          В системе отчёта первого близнеца координата обоих близнецов должна быть одинаковая в момент возвращения, иначе они не смогут встретиться.


          1. zuborg
            02.10.2017 15:50

            Ну она и будет одинаковой, 50лет 0дней 0секунд например — это момент встречи в системе координат летавшего близнеца.


      1. Hanako_Seishin
        04.10.2017 17:27

        Так он ведь не через пятое измерение в новое t попадает. Каждый из близнецов существует в каждом моменте времени t (по крайней мере от своего рождения до смерти), если в один из таких моментов и их пространственные координаты совпадают — это и есть встреча. Т.е. например, улетел близнец с Земли в 2000 году, полетал годик-другой, и вернулся уже в 2020. Но все прошедшие на Земле 20 лет близнец второй близнец так и сидел на Земле и никуда с неё не делся. То есть они встречаются в координатах t=2020, x=0, y=0, z=0 (в системе отсчёта Земли).


  1. Hemeg
    01.10.2017 01:06
    +1

    Лучше бы написали нам дельную статью про эффект Зенона, было бы круто.
    Или дали популярный перевод статьи Верлинде, где он выводит эффект темной материи на основе эмерджентной гипотезы и КВК, с указанием его возможных ошибок. Я например смог понять только общую канву, знаний не хватило. Ну или хотя бы про энтропию черных дыр, думаю многим будет интересно, почему энтропия зависит не от объема, а от площади поверхности.
    А то все научно-популярную статьи про современную физику в рунете, это перепосты с западных сайтов, да еще и плохо переведенные.


    1. Tyusha Автор
      01.10.2017 01:12
      +6

      LOL:

      Лучше бы написали… почему энтропия зависит не от объема, а от площади поверхности

      Здесь уместно привести физтеховский анкедот:

      «Над этим вопросом сейчас бьётся вся наша кафедра. Если решим, включим во вступительные экзамены».


  1. PretorDH
    01.10.2017 02:18
    -3

    0 — «НОЛЬ» — Расстояние между кораблями в системе отсчета корабля.

    И если они на встречном курсе, то красивые релятивистские эффекты гарантированы.

    На вскидку, пофантазирую:
    Наблюдатель с Земли на протяжении 4 лет будет наблюдать как корабль растягивается в длину. А потом на половине пути яркая вспышка двух аннигилирующих кораблей. А дальше

    На корабле произойдет другое: в следующую секунду после набора скорости передние части кораблей начнут аннигилировать, поглощая кинетическую энергию и придавая отрицательное ускорение «a» в формуле (5) кораблям. Расстояние и время теперь растянутся. А сам процесс аннигиляции по времени корабля займет примерно 4 года. При этом весь корабль не аннигилирует. Аннигилирует только определенная часть материи кораблей энергетически равная кинетической энергии израсходованной на ускорение… Корабль конечно развалится на атомы.

    Самое интересное, что для наблюдателя на корабле по первой, не будет даже вспышек. Со временем интенсивность излучения от аннигиляции будет увеличиваться. В финале Биг-бада-бум.

    Но вот если например, два корабля выйдут на встречные курсы и их скорость сближения будет выше скорости света (это ещё надо постараться, с ходу даже не соображу как). Тогда наблюдатель на корабле будет наблюдать картину медленно и неумолимого объединения воедино двух кораблей на «квантовом барьере» пятого измерения.
    Предсказать странный финал такого действа. Возможно корабли просто пройдут сквозь друг друга. Возможно распределяться по измерениям, в каждом из которых будет химера. Но это будет сопровождаться гравитационным или нейтринными всплеском «невиданной» красоты.


    1. Tyusha Автор
      01.10.2017 02:23
      +2

      Уже поздно, шли бы поспать.


  1. ideological
    01.10.2017 04:10

    Рад что тут эксперты собрались, объясните пожалуйста мне тапку про парадокс близнецов и все фишки со временем:
    1.) >>путешественник окажется моложе своего брата-домоседа
    Это его собственные впечатления будут? Ведь если посмотреть со стороны — один брат улетел и прилетел, другой сидел на месте, мы наблюдаем это процесс и время вроде как прошло одинаковое :)
    2.) Когда говорят что время зависит от гравитации и «замедляется/увеличивается» — имеют ввиду механические часы и принцип их работы?
    3.) Почему вообще время ассоциируют со скоростью света? По тапкологике время лишь абстракция, просто замер от одних событий до других. И получается что всякие там путешествия и ускорения не создадут нам машину времени.
    4.) В Интерстелларе персонажи были на планете где одна минута это 100500 времени на Земле. Это они так медленно так двигались :)? Или чтобы увидел наблюдатель между землей и тормозопланетой?
    (я знаю что это просто фильм и мне кстати не понравился; просто пишут научно-фантастический :))

    Это не троллинг и прошу не минусовать (это вопрос ребята, не все тут физики же), вся моя палата хочет объяснений. Пожалуйста.


    1. tishevich
      01.10.2017 10:12

      1) нет, это будут объективные впечатления. Т.к. один из братьев будет на самом деле моложе, потому что на корабле время шло медленнее.
      2) нет
      3) потому что в нашем пространстве время это физическая величина, как ваш стул и стол, только потрогать нельзя и оно связано со скоростью, и все вместе с расстоянием и гравитацией. Машина времени возможна, но только в будущее.
      4) нет, они двигались нормально т.к. пребывали в своем локальном времени. В разных частях пространства время может быть разным. Наблюдатель бы увидел как они целый год делают один шаг.


      1. Tyusha Автор
        01.10.2017 12:49
        +2

        1. Парадов чуток не в этом. А в том, кто именно будет моложе в итоге. Брат на Земле может сказать: у тебя на корабле часы идут медленнее. Но брат-путешественник может возразить: да нет же, это у тебя время замедлилось. (Мы же можем анализировать ситуацию из его системы отсчёта). Так кто прав? Кто и итоге будет моложе? Путешественник может также рассуждать, что это не он, а землянин сначала улетел от него, а потом приблизился обратно, поэтому моложе должен быть именно землянин! Парадокс в этом, а не в том, что время просто замедляется.

        Решение в том, что правы оба до тех пор, но только пока движутся равномерно. Но для полёта туда и обратно путешественнику нужно где-то ускоряться. Суть в том, что это делает ситуацию несимметричной. Поэтому можно опираться только на рассуждения землянина, т.к. он оставался в инерциальной СО.

        Можно сделать расчёт и с точки зрения брата-путешественника. Но это уже сложно математически. Но если его проделать, то получится тот же результат — он вернётся более молодым. Парадокс снимается.


        1. michael_vostrikov
          01.10.2017 21:05

          А подскажите, раз время при движении замедляется, то можно ли считать, что скорость абсолютна, а не относительна?


          1. Tyusha Автор
            01.10.2017 21:06

            Нет. Скорость относительна.


            1. michael_vostrikov
              01.10.2017 22:09

              Почему тогда при движении ракеты относительно Земли часы на ракете замедляются по отношению к часам на Земле? Земля ведь относительно ракеты тоже движется с такой же скоростью. Ок, пусть не скорость, а движение как таковое. В чем различие между этими двумя телами?


              1. Tyusha Автор
                01.10.2017 22:21
                +2

                Отличий никаких нет. Обе системы отсчёта равноправны, и рассуждения можно проводить в любой их них. С точки зрения ракеты наоборот часы на Земле идут медленнее. В этом есть некоторый Парадокс близнецов. Я его даже тут объясняла уже. Или почитайте в Интернете, это же известный вопрос.

                Если оба движутся, то чтобы «сверить часы» им надо перейти в одну систему отсчёта, т.е. кому-то придётся остановиться (вернее изменить свою скорость, чтобы сравняться с другим). И тут прикол в том, что тот кто остановится — тот «проиграл», т.к. он побывал в неинерциальной системе отсчёта, когда двигался ускоренно (замедленно).


                1. michael_vostrikov
                  01.10.2017 23:03
                  -1

                  Но подождите, если считать что ракета неподвижна и движется Земля, то при остановке ракеты получается, что замедлялась Земля. То есть ускорение было у Земли и это она была в неинерциальной системе отсчета. Так?


                  В интернете я читал, но там вопросы некому задать.


                  1. Tyusha Автор
                    01.10.2017 23:13
                    +2

                    Пока летят равномерно, обе системы равны. Но вот кто именно стал замедляться, определить всегда можно, т.к. система перестаёт быть инерциальной и движется ускоренно. Ускорение можно всегда заметить по возникновению силы тяжести. И именно это выделяет ракету, в этот момент симметрия ситуации нарушается. Теперь можно точно сказать, кто движется равно, а кто нет.


                    1. michael_vostrikov
                      02.10.2017 00:28

                      Хм, на Земле постоянно присутствует сила тяжести. Как быть с ней? Даже если это не учитывать, допустим, близнец-путешественник тормозит с ускорением 1g, а близнец-домосед сел в ракету и попеременно то ускоряется то тормозит с тем же ускорением. Таким образом, ускорение у них присутствует, но второй далеко от Земли не улетит. Но ведь все равно первый будет моложе?


                      Кроме того, почему вообще в ракете возникает сила тяжести? Для относительной остановки ракеты можно либо ракету замедлить, либо окружающий мир разогнать. Относительный результат одинаковый, но сила тяжести проявится в разных телах. Получается, где-то есть абсолютность, которая и определяет где именно проявится? Или нет?


                      1. Tyusha Автор
                        02.10.2017 01:48
                        +1

                        {facepalm} Что-то вы так всё напутали, что я даже не знаю как подступиться.

                        1. Под «Землёй» имеется ввиду неподвижная точка отправления, сила тяжести на Земле не рассматривается. Считайте, что её нет.

                        2. Сила тяжести возникает в ракете, когда она движется в космосе ускоренно, даже вдали от гравитирующих тел.

                        3. Если разгонять не ракету, а окружающий мир, результат будет другой. Относительность работает только для двух инерциальных систем, когда нельзя выделить предпочтительную систему. Когда одна система инерциальная, а другая неинерциальная, то никакой относительности уже нет. И физика в двух системах становится разной.

                        4. Оставшийся брат близнец может летать на Земле туда сюда. Да действительно он может в таком случае стареть меньше, чем если бы сидел неподвижно. Он может даже так метаться туда-сюда, что окажется даже моложе вернувшегося брата.


                        1. michael_vostrikov
                          02.10.2017 14:10

                          1. Но ведь она есть. Почему мы рассматирваем силу тяжести в ракете, но не рассматриваем на Земле? Природа-то у них одна.
                          2. Да, я это и подразумевал.
                          3. Но пока ракета летит равномерно, они же обе инерциальны? Получается, нет разницы, разгонять одну или тормозить другую. Ок, можно вместо мира взять другую ракету. Конечный результат один — ракеты неподвижны относительно друг друга, но сила тяжести в зависимости от варианта будет в разных ракетах.
                          4. Речь о том, что он будет "метаться" с тем же ускорением, что и брат (по модулю). Только не будет разгоняться до большой скорости. То есть, они оба побывают в неинерциальных системах отсчета, и в этом разницы между ними нет.


                          1. itlen
                            02.10.2017 14:36

                            1. Нет не одна. У ускорения и гравитации схожие эффекты. Если топнете газ на машине, вас вдавит в кресло. Но это же не значит что машина создала гравитационное поле.


                            1. Hardcoin
                              02.10.2017 22:55

                              А как их отличить? Я намекаю на принцип эквивалентности.


                              1. itlen
                                03.10.2017 11:35

                                Если провести чистый эксперимент то никак не отличите находитесь ли вы в кресле на Земле или движетесь в кресле по орбите с ускорением g.


                                1. Druu
                                  03.10.2017 12:16

                                  Только в том случае, если вы — точка. А на объем будут, как минимум, действовать приливные силы.


                            1. michael_vostrikov
                              03.10.2017 07:41
                              +1

                              Не ускорение, а инерция. В общем-то должно быть наоборот — если машина создаст гравитационное поле, то вас в кресло не вдавит, так как на вас оно тоже будет действовать. Если она будет таким полем тормозить, будет ощущение как будто просто лег на спину или падаешь назад.


                          1. alexzzam
                            03.10.2017 17:44
                            +1

                            1. В данном случае эффект не из-за силы тяжести. Просто системы разные. Ну представьте тот же парадокс близнецов, только два корабля А и Б в космосе. Б ускорился, полетал, вернулся и затормозил, в Б прошло меньше времени. Они неравноправны, мы их можем различить, потому что:
                            3. Когда один корабль испытывает ускорение, а второй нет, мы прямо точно знаем, где какой, относительности нет. Например, в том, у которого ускорение, появляется сила тяжести в сторону, противоположную ускорению.
                            4. Разница есть. Потому что это не маркер такой «бывал в неинерциальной системе», а история. Нужно для аккуратного подсчёта все его метания учесть. Сколько секунд куда ускорялся и сколько у него локального времени при этом прошло.


                            1. michael_vostrikov
                              03.10.2017 21:57

                              Ну то есть можно отличить, что эта ракета движется в пространстве относительно вон той, а та стоит на месте, а не наоборот?


                              Сколько секунд куда ускорялся и сколько у него локального времени при этом прошло.

                              Так неважно, я же сказал что он далеко не улетал, его скорость небольшая, значит замедлением можно пренебречь. Я о другом. Замедление же зависит не от ускорения, а от скорости. Про обе ракеты можно сказать, что одна движется относительно другой, но после остановки замедление проявится только в одной из них. И если не останавливать первую, а разогнать вторую, то близнец в первой все равно будет моложе, так как он летел дольше. Так? Значит есть способ определить факт движения в пространстве, то есть относительность мнимая. А если нет, то что я не учитываю?


      1. ideological
        01.10.2017 16:22

        4.) >>В разных частях пространства время может быть разным.
        Несходец логический. Получается есть более и менее крутые по времени планеты. И человек на родной тормозопланете может улететь на быструю, там протусить, стать умняшкой и вернутся к своей тормозной, где за это время успели только сделать пару шагов.
        И это кажется странным. Товарищ ниже тоже в шоке:
        «Если разница будет, то получается на каждой из планет во вселенной время идет по разному что на мой дилетантский взгляд как-то странновато.»
        Точно-точно на разных планетах время может быть разным?


        1. tishevich
          01.10.2017 20:07

          Точно-точно на разных планетах время может быть разным?


          Конечно. На планете вблизи черной дыры время будет другим за счет искривления пространства, потому что планета находится в гравитационном колодце. Это если на двух пальцах объяснять.


        1. Hardcoin
          02.10.2017 22:59

          Может, конечно, а в чем проблема? С интуицией не сходится, потому что опыта помещениям других планет пока нет.


          Проблема только в том, что гравитация на тормозопланете крайне велика. Выжить будет тяжело


        1. michael_vostrikov
          03.10.2017 07:52

          Не стоит судить по Интерстеллару. Чтобы была такая разница во времени, сила гравитации должна быть очень большой, почти как у черной дыры. Их бы сразу притянуло и расплющило вместе с ракетой.


          1. Tsimur_S
            03.10.2017 13:21

            Вы внимательно смотрели? Сила гравитации была такой большой потому что она в самом деле происходила от черной дыры. У самой планеты сила притяжения была 1,3g. Почему такая мощная гравитация не стащила планету с орбиты это уже другой большой вопрос, скорее всего она должна нарезать круги вокруг этой дыры с околосветовой скоростью что бы хоть как то её уравновесить.


            1. michael_vostrikov
              03.10.2017 14:12

              Да, точно. Помню что тоже тогда про скорость подумал. Но суть не меняется. При такой силе тяжести они не смогли бы произвольно передвигаться, их бы притянуло в направлении этой силы. Не на планету так на саму дыру.


              1. Druu
                03.10.2017 17:15

                > При такой силе тяжести они не смогли бы произвольно передвигаться, их бы притянуло в направлении этой силы.

                Сила тяжести сама по себе никак передвигаться не мешает, т.к. действует и на планету, а приливные силы вблизи больших чд очень слабые (настолько, что живой космонавт без проблем сможет пролететь сквозь горизонт событий).


                1. michael_vostrikov
                  03.10.2017 22:13

                  Насколько я это представляю, она не мешает, пока передвигающийся находится в свободном падении. Если падению будет что-то мешать, например палуба корабля, улетающего в противоположном направлении, то возникнет перегрузка.


                  1. Druu
                    04.10.2017 00:56

                    > Если падению будет что-то мешать, например палуба корабля, улетающего в противоположном направлении, то возникнет перегрузка.

                    Перегрузка будет вызвана движением корабля и будет одинаковой в любом гравитационном поле. А так — планета и идущий по ней человек находятся в свободном падении в гравитационном поле ЧД и никак друг другу не мешают.


                    1. michael_vostrikov
                      04.10.2017 10:24

                      Почему одинаковой? Если ускорение падения 10g, то для движения в другую сторону нужно больше 10g. С Луны взлетать легче, чем с Земли.


                      Ну я и говорю, пока они падают в направлении черной дыры, передвигаться можно. А если планета в таком поле движется вдоль поверхности ЧД, то это как на самолете лететь. Сила тяжести ЧД все равно будет ощущаться.


                      Причем силу тяжести самой планеты можно чувствовать, либо если поле ЧД меньше поля планеты (то есть планета далеко от нее), либо если космонавт находится на той части планеты, которая дальше от ЧД (то есть вектора сил складываются).


                      1. Druu
                        04.10.2017 14:19

                        > Почему одинаковой? Если ускорение падения 10g, то для движения в другую сторону нужно больше 10g.

                        Но эта перегрузка будет вызвана движением корабля. И совершенно неважно, будет двигаться корабль в гравитационном поле или нет — она зависит _только от ускорения корабля_. Если корабль движется неускоренно (находится в свободном падении на стационарной орбите), то никаких перегрузок и не будет.

                        > Причем силу тяжести самой планеты можно чувствовать, либо если поле ЧД меньше поля планеты (то есть планета далеко от нее), либо если космонавт находится на той части планеты, которая дальше от ЧД (то есть вектора сил складываются).

                        В ОТО инерциальной системой является любая система, которая находится в состоянии свободного падения. Если вы с планетой находитесь в состоянии свободного падения в ЧД, то нет никакого способа отличить эту ситуацию от простого полета в космосе вдали от ЧД или вообще любых гравитационных полей (да, в реальности ваш объем с планетой ненулевой, так как вы не материальная точка, а значит есть приливные силы и эффекты нелинейности, но они пренебрежимо малы).

                        По-этому наличие рядом ЧД никаким значимым образом на ваше взаимодействие с планетой не влияет, как ходили вы по ней спокойно, так и будете.


                        1. michael_vostrikov
                          04.10.2017 19:07

                          Почитал информацию, наверно вы правы. Я представлял случай когда корабль не движется вбок и пытается сохранить расстояние. А если движется с достаточной скоростью, то расстояние и так сохраняется, я это как-то упустил, хоть и знаю про спутники.


            1. sumanai
              03.10.2017 17:04

              Скорее всего её расфигачит приливными силами на красивый светящийся аккреционный диск.


    1. El_bruja_de_la_tristefigu
      01.10.2017 12:14
      +1

      Парадокс близнецов объясняется тем, что брат-космонавт все это время находился не в инерциальной системе отсчета, а двигался ускоренно — в этом месте задача теряет симметричность. А почему при ускорении время идет медленнее обьясняет уже ОТО.


      1. Tyusha Автор
        01.10.2017 13:02
        +2

        ОТО здесь не причём. СТО замкнутая теория, и между 1904 и 1915 годами, пока не было ОТО, учёные должны были прекрасно справляться с парадоксом близнецов, включая расчёт в ускоренной системе.


    1. itlen
      01.10.2017 12:16
      +1

      Почему вообще время ассоциируют со скоростью света?

      Потому что мы живем не просто пространстве, а в пространстве Минковского

      Чтобы понять что время далеко не просто абстракция для удобства, а вполне себе физическая метрика нашего пространства времени представьте ситуацию: вы договорились о встрече с кем-то. Встреча в торговом центре ну углу улиц N и M (x и y координата), на 5 этаже (z координата), сегодня в 17:00 (t координата). Уберите любую координату и встречи не будет. Получается что они все равноценны.


      1. Hardcoin
        01.10.2017 12:39

        Если я переместился по координате t на 5 часов вперёд, а вы, полетав с ускорением и вернувшись, на 10 минут, то как мы встретились? У нас вообще одинаковая t-координата или разная, когда вы вернулись на то же место?


        Не троллинг, реально интересно.


        1. itlen
          01.10.2017 13:17
          +1

          t-координата это конкретное время в конкретной системе отсчета — 17:00 — она не может быть разной в этой системе ни для кого.Попасть в нее из координаты 16:59 мы можем с разной скоростью: я с С/2 на орбите за 10 минут (по моему времени), вы с околосветовой на пути к Альфа Центавре и обратно за 5 часов.

          Когда вернемся (через минуту в той системе где договорились встретится, допустим нас на Земле ждет третий друг) на моих часах будет +10 минут, на ваших +5 часов, на часах нашего друга +1 минута. На земле будет 17:00, но достигли мы этой координаты за разное время. Я прожил на 10 минут дольше относительно нашего третьего друга на Земле, вы прожили на 5 часов дольше относительно него же, относительно меня вы прожили не на 5, а скажем, на 4,5 часа дольше. Это надо уже считать.


          1. ideological
            01.10.2017 16:34

            Ещё один дилетанский вопрос. А практически такое проверялось уже? Ну можно же сделать какие-нибудь миниспутники с разной скоростью по разным траекториям заставить с замеренными часами встретится в одной точке около земли? Нечто такое уже было? Как проверить что это не просто ошибочная математическая запись несовместимая с реальностью?


            1. itlen
              01.10.2017 21:05
              +3

              Конечно Эксперимент Хафеле — Китинга.

              Более того эффекты замедления времени учтены в спутниковых навигационных системах. Например спутник ГЛОНАСС движется по своей орбите со скоростью ~3,75 км/с и его релятивистская поправка 4,36х10?10, то есть 37,7 мкс в сутки. Каждые сутки он на 0,000037 секунды перемещается в будущее. Для спутника GPS, который движется по более высокой орбите с более высокой скоростью это время составляет 45 микросекунд, если не ошибаюсь.

              Так что не нужны никакие эксперименты чтобы это подтвердить — это реальность. Не будь этих поправок ваш навигатор в машине бы не работал корректно.


              1. ideological
                01.10.2017 22:07

                Понятно. Спасибо.

                А атомные часы на земле кто-нибудь пробовал умышленно замедлить? Например внешним воздействием какого-нибудь излучения? Такие эксперименты ведутся?

                Замедление атомных часов из-за гравитации очевидно. Но тут опять же можно сказать что это просто косяк в замерах. Такова конструкция часов. А время проходит столько же :). Как же доказать что именно время как величина замедляется или ускоряется?


                1. Tyusha Автор
                  01.10.2017 22:25
                  +1

                  «Умышленно замедлить» можно применив какой-то физический эффект, какое-то воздействие. Это будет принципиально другое замедление, я бы сказал, физическое. В то время как гравитационное замедление времени, это фундаментальное замедление. Причём только с точки зрения другого наблюдателя, находящегося вне этого поля. Для того, кто находится внутри, все процессы идут как обычно.


                1. vedenin1980
                  01.10.2017 22:27

                  Не время как величина, а все физические процессы происходят быстрее или медленнее. К сожалению, время само по себе не возможно измерить. Но кроме часов там еще проводились эксперименты с красным смещением излучения на разной высоте, которые дали тот же результат. Можно сказать что все эксперименты говорят что все процессы происходят быстрее или медленнее, что аналогично замедлению или ускорению времени.


                1. itlen
                  01.10.2017 23:25

                  Ниже ответил.


            1. Druu
              01.10.2017 21:06

              > Как проверить что это не просто ошибочная математическая запись несовместимая с реальностью?

              Здесь есть важный момент — этой модели требуют у-я Максвелла (преобразования Лоренца, лежащие в основе СТО, выводятся из у-й Максвелла как преобразования, которым, условно, должна удовлетворять структура пространства-времени, в которой могут непротиворечиво происходить описываемые у-ми Максвелла процессы) и, т.о., вся электро-динамика. То, что электро-магнитные явления работают так, как работают (в согласии с у-ми Максвелла) и есть самое что ни на есть первейшее подтверждение СТО. Что до спутников — в тех же GPS/Глонасс поправки как СТО так и ОТО необходимо учитывать (либо сразу, либо за счет регулярной коррекции) и это делается, иначе указанные системы просто не смогут работать.


            1. dmitry_dvm
              01.10.2017 21:31

              Вроде как в gps это в хвост и в гриву применяется.


              1. IvanKor2017
                01.10.2017 22:07
                +1

                Уже показал что в GPS ни того ни другого не применяется.


                1. Druu
                  02.10.2017 14:25
                  +1

                  Так речь не о частоте сигнала, а о часах. Если их не подводить (или не внести поправки в сами часы, или не учитывать поправки при расчете), то приемник будет получать все более и более смещенный сигнал (за несколько лет там получается что-то порядка секунды), а время прохождения сигнала рассчитывается именно по смещению.


            1. Bhudh
              01.10.2017 22:18
              -1

              Братья Келли сойдут за экспериментаторов/участников эксперимента?


            1. arheops
              01.10.2017 23:06

              Конечно, почитайте про GPS. Там настолко малы интервалы времени, что вводится поправки на скорость спутников.


          1. Hardcoin
            01.10.2017 17:02

            Интересное объяснение, спасибо.


            Попасть с разной скоростью — вы имеете ввиду единицу измерения секунд в секунду? Или в чем измеряется эта разная скорость попадания в координату 17:00?


            1. itlen
              01.10.2017 23:09
              +1

              Да секунд в секунду. Это контринтуитивные вещи такие себе. Чтобы понятнее стало вот классическая аналогия: две машины едут в пункт назначения строго на север со скоростью v=100 км/ч. Соответственно к пункту назначения они приближаются со скоростью V'=v=100 км/ч. На развилке одна сворачивает на другую дорогу и начинает двигаться строго на северо-восток с той же собственной скоростью v. Тут уже интуитивно понятно что ее скорость приближения к пункту назначения, V', стала меньше, ее «отобрало» движение на восток. Наблюдатель первой машины видит как вторая начала терять в своей скорости V', замедляться в приближении к пункту назначения, наблюдатель во второй машине видит как скорость приближения к пункту назначения (та же V') для первой, стала быстрее.

              Теперь представьте двумерную систему координат где классическая ось Y это время, классическая ось X это пространство (x,y,z). Получится пространство Минковского метрики 3.1. Заменим также собственную скорость машины v на с=скорость движения во времени (секунда в секунду), а скорость приближения к пункту назначения V' на C' — скорость приближения к будущему — тоже секунда в секунду. Заменим машины на две инерциальные системы отсчета, а пункт назначения на координату по оси Y. Так как по этой координате время, то зададим ее, например t=01.10.2018 00:00. Через год.

              Получается что две неподвижные инерциальные системы отсчета движутся во времени с одинаковой скоростью (по оси Y со скоростью с). Вдруг одна система начинает движение в пространстве (по оси X, точнее по любой из (x,y,z) которые лежат на оси X в пространстве Минковского, «на восток»). Собственная скорость систем (с) не меняется, но скорость приближения к t становится разной. Они начинаю двигаться с разной скоростью C'. А что такое у нас C'? Скорость приближения к пункту назначения, к координате t, к будущему.

              Они движутся во времени с разной скоростью!


    1. andrey1111
      01.10.2017 12:16
      +1

      Время — это скорость протекания процессов.
      С точки зрения СТО время, то есть скорость протекания всех процессов, при движении замедляется. Это не кажущийся эффект, космический близнец после возвращения будет реально моложе земного.
      Не важно какими часами мы меряем. Скорость движения влияет на все процессы, механические часы более сложно устроены как система, чем атомные. Поэтому они менее точные — гораздо больше плохо контролируемых факторов влияют на их точность.
      Скорость света является глобальным инвариантом, поэтому через нее пересчитывают время и расстояния в локальных системах отсчета.
      Ускорение не влияет на замедление и ускорение времени, это видно из формул СТО.


  1. Hardcoin
    01.10.2017 04:18

    2) имеют ввиду любые часы, в т.ч. атомные. На спутниках gps как раз такие, там, говорят, вносят поправку в соответствии с теорией относительности.


    1. IvanKor2017
      01.10.2017 14:36
      +1

      На спутниках gps просто выставили вместо 10.23 MHz частоту 10.22999999543 MHz т.е. разница составляет 0.00457 Hz, это ничтожно малая, экспериментально не обнаруживаемая величина, ибо за счет эфф. Доплера частота 10.23 MHz всегда плавает в пределах ~ ± 2500 Hz, т.е. эфф. Доплера более чем на 5 порядков более существен. + за счет ионосферы частота плавает ~ ± 1 Hz (проверяется при помощи геостационарной SBAS, вспомогательной GPS системы ).
      Часто в жёлтой прессе вместо 0.00457 Hz упоминают 5 Hz рел. эфф. что является вымыслом.


      1. Hardcoin
        01.10.2017 15:33

        Вы имеете ввиду, что сдвиг на пять миллигерц не обязателен?


        Эффект Доплера конечно учитывается тоже.


        1. IvanKor2017
          01.10.2017 16:17
          +2

          Эффект Доплера учитывается первым делом. Собственная долговременная нестабильность рубидиевого стандарта один миллигерц, кратковременная будет больше чем пять миллигерц релят. поправки. Время на приемной стороне это сигнал с NCO приемника, частота которого обычно гуляет где то в пределах 10 Гц или в особо точных 1Гц. Сдвиг ОДНОВРЕМЕННО на всех спутниках в пределах 5 мГц это ничто, естественно никто это не учитывает. Правильно, по теории ОТО, необходимо вычислять и изменять relativistic = -4.442807633E-10*eph.ecc*eph.sqrta*sek;


  1. DancingOnWater
    01.10.2017 09:00
    +2

    Тут надо вспомнить, о лоренцевском сокращении длины. Ведь разогнавшись до половины скорости света мы обнаружим, что масштаб вдоль направления нашего движения сжался.

    Эм, нет. Лоренцево сокращение — это сокрашение движущейся длины. Т.е. относительно Земли видимое ваши размеры уменьшились. Метрическое изменеие — это уже ОТО, это уже гравитация.

    Ну и наконец, относительно движущегося коробля его скорость равна нулю.


  1. CrazyRoot
    01.10.2017 09:05
    -1

    Где то я похожее видел уже…

    Вспомнил!
    image


    1. justhabrauser
      01.10.2017 10:50
      +2

      Эть… Предпоследнее шаманство некорректно.
      Зрители заметили ниточку, учите дальше «ловкость рук» :-)


  1. Tertium
    01.10.2017 11:15
    +1

    В статье обясняется принцип действия кораблей из «Билла — героя галактики» видимо. Математику применять к физике дело ответственное, так можно и до физ. смысла комплексных корней квадратного уравнения дойти, и до голографического принципа.
    А по количеству комментов видно, на что люди возбуждаются больше всего — на девушек и псевдо-около-вроде-науку. То же рен-тв всегда включает в передачи оба аспекта.


  1. Tertium
    01.10.2017 11:22
    -1

    Девушка — тролль… Нет, не так. Девушка — достаточно тонкий тролль. Нет, так тоже нельзя. Это же девушка, а троллинг — достаточно тонкий. Какие еще есть народности… Девушка — темный эльф?..


  1. ReakTiVe-007
    01.10.2017 11:33
    -1

    А техническое решение так и не обсудили image


    1. IvanKor2017
      01.10.2017 13:30
      +1

      1. darthmaul
        01.10.2017 22:15
        +1

        Кстати, есть какие — нибудь новости от них? С одной стороны от них веет фричеством, а с другой — это всё же НАСА.


        1. IvanKor2017
          02.10.2017 12:11

          Писали что опыт с интерферометром провели более чем успешный.


          1. darthmaul
            02.10.2017 12:52

            А не подскажете что о них можно почитать? Видел статейку якобы оно пересчитали расчеты Алькубьерре и смогли "обойтись" вполне земными объемами энергии (вроде как сто реакторов авианосца достаточно) для своего двигателя. Но расчётов там не обнаружил.


  1. SkyMind
    01.10.2017 11:49

    А вся эта чехарда со временем и пространством связана со скоростью или же с ускорением?
    К примеру если каким-то чудесным образом телепортировать часы в далекую-далекую галактику, удаляющуюся от млечного пути со скоростью, близкой к скорости света, а затем вернуть эти часы обратно тем же волшебным способом, то будет ли разница в показаниях тех часов, что оставались на земле и в тех, что были телепортированы туда-сюда?
    Если разница будет, то получается на каждой из планет во вселенной время идет по разному что на мой дилетантский взгляд как-то странновато.
    А если разницы не будет, то ответ на вопрос задачки — расстояние между кораблями как со стороны внешнего наблюдателя, так и со стороны экипажа корабля будет одним и тем же — 4.36 световых года (расстояние между землей и альфа центавра). Ведь в условии задачи указана только скорость (ускорение как бы было мгновенным).


  1. gaploid
    01.10.2017 12:30

    Никогда не понимал почему нельзя разогнать ракету до полу-световой скорости, а с нее запустить еще одну. И так метрешычным способом разгонять до чего хочешь.


    1. Hardcoin
      01.10.2017 12:35
      +2

      Почему нельзя? Можно. Между двумя ракетами вполне может быть с/2. Вот только у второй ракеты с землёй будет не с, а меньше.


    1. vedenin1980
      01.10.2017 12:37
      +3

      Можно, только все равно не поможет. При релятивистском движении скорости складываются не по v1 + v2, а несколько сложнее.


  1. Lorents
    01.10.2017 12:43

    У меня давно есть вопрос. Земля, Солнце, наше галактика фактически имеют свою скорость движения.

    Если использовать преобразование Лоренца:
    image

    Получается у нас уже на Земле происходит «искажение» времени.


    1. voyager-1
      01.10.2017 12:55
      +3

      Да, но очень немного — за всё время существования Земли часы ушли на 3650 лет (эффект только от скорости относительно реликтового ускорения). Ещё 3 года к этому добавляет нахождение в гравитационном колодце Земли. Подробнее эти эффекты я здесь расписывал.


      1. Tyusha Автор
        01.10.2017 13:09
        +3

        О, замечательно! Как-то пропустила этот пост. Но товарищ на самом деле спрашивал о другом, подразумевая не гравитацию и не ускоренное движение по орбите, а про равномерное. Ответ в том, что нет выделенной инерциальной системы, все они хорошо. Каждый может с полны правом сказать, что у него со временем всё в порядке.


    1. vedenin1980
      01.10.2017 12:59
      +2

      Происходит и даже гравитация Земли и Солнца тоже «искажают» время, другое дело на сколько. Скорость Земли относительно реликтового излучения около 368 км/с, скорость света 299 792 км/с, если подставить значения в формулы окажется (если я все правильно посчитал) что время искажено на 0,00015%.


      1. vedenin1980
        01.10.2017 13:07
        +2

        Чтобы было понятно за год разницы во времени с реликтовым излучением набегает (по мои подсчетам) что-то около минуты.


  1. Hemeg
    01.10.2017 15:10

    Tertium
    01.10.17 в 11:15

    0

    В статье обясняется принцип действия кораблей из «Билла — героя галактики» видимо. Математику применять к физике дело ответственное, так можно и до физ. смысла комплексных корней квадратного уравнения дойти, и до голографического принципа.


    Так и представляю себе мостик космического корабля:
    Капитан: Увеличить скорость!
    Навигатор: <Беря в руки карандаш и лист бумаги> Делю расстояние на время!
    Капитан: Еще!
    Навигатор: Повторяю процедуру!
    Капитан: Быстрее!
    Навигатор: На Сэр! У меня кончится грифель, мы не сможем затормозить!


  1. ariksu
    01.10.2017 18:39

    Извините, я у вас уже на второй формуле сломался, вы не особо их поясняете. Что такое у вас dL/dt и чем они отличаются от deltaL/deltat? То есть написано «к скорости (относительно чего?) добавляется динамическое сокращение пути (с чего бы вдруг к скорости добавлять динамическое изменение пути, которое и есть скорость) », но я никак не смог завязать эти слова и эту формулу.


    1. Lennonenko
      03.10.2017 01:24

      дельта — это какое-то конечное изменение, deltaV=V2-V1
      тогда как «d» — символ дифференцирования, взятия производной
      dL/dt (дэ-эль по дэ-тэ) — изменение функции при изменении аргумента, стремящемся к нулю, данном случае, изменение расстояния со временем, L здесь — общее расстояние до выбранной конечной точки
      сам факт наличия скорости изменяет преодолеваемое расстояние L, то есть делает скорость ещё скоростнее
      в итоге для фотона субъективная скорость бесконечна, а субъективное время равно нулю
      выше уже заметили, что поговорка «тише едешь — дальше будешь» обретает в этом свете новый смысл


  1. AlexeyVPolunin
    01.10.2017 18:51

    Спрошу автора статьи. Если бы, например, человечество было бы слепым и ориентировалось только за счет слуха, то максимальную скорость в природе приняли бы за скорость звука?


    1. sumanai
      01.10.2017 20:11

      Нет конечно же, изобрести электромагнитные приборы и измерить скорость эм-волн, то есть того же света, можно и слепыми.


      1. AlexeyVPolunin
        01.10.2017 20:17

        А откуда слепые люди узнали бы о существовании света?


        1. tmnhy
          01.10.2017 20:18

          Инфракра?сное излуче?ние.


          1. AlexeyVPolunin
            01.10.2017 20:26

            Это тепло. Инфракрасное излучение видит зрячий. Слепой его ощущает как тепло.
            ИМХО. Скорость света была выбрана в качестве максимально возможной, так как из наших органов чувств наиболее «быстродействующим» является зрение. Все наши измерительные приборы привязаны к нашим органом чувств, иначе они становятся бесполезными.
            Косвенно это подтверждает эффект Вавилова-Черенкова.


            1. vedenin1980
              01.10.2017 20:37

              Скорость света была выбрана в качестве максимально возможной, так как из наших органов чувств наиболее «быстродействующим» является зрение

              Она не выбрана, а формулы (весьма неочевидные, кстати) и эксперименты показали, что она максимально возможная. А скоростью звука такого бы не было, даже если мы вообще не открыли другие излучения.

              наши измерительные приборы привязаны к нашим органом чувств,

              нет, прибор выдает результат адаптированный часто для наших органов, но измерять он может что-то никак не близкое к нашим органам чувств (квантовые эффекты или гравитационные волны). Если бы существовало излучения более быстрое, чем свет приборы бы его нашли просто измерив скорость из точки А в точку В.


              1. AlexeyVPolunin
                01.10.2017 20:43

                Я не спорю, я говорю ИМХО. Но все же, попробуйте разобраться в эффекте Вавилова-Черенкова. Затем из жидкостной среды мысленно перенесите эксперимент Черенкова в вакуум и спросите себя, если в одной среде процесс, происходящий со скоростью света порождает процесс, происходящий со скоростью, превышающей скорость света в той среде, то почему же невозможны процессы, происходящие со скоростью, превышающей скорость света в вакууме?
                Вполне возможно, что многие из этих процессов мы наблюдаем, адаптировав их под наши органы чувств. Что-то вроде гетеродина в радиоприемнике. Но это лишь искаженный отпечаток реального процесса, о котором мы не знаем.


                1. Lennonenko
                  03.10.2017 01:32

                  в эффекте черенкова скорость движения частиц в среде превышает скорость света в среде
                  в вакууме это невозможно
                  давным-давно мир познаётся не зрением, а разумом, путём построения разного рода абстракций, в рамках которых объясняется тот или иной феномен


                  1. AlexeyVPolunin
                    03.10.2017 07:34

                    в вакууме это невозможно

                    Вы вводите начальное условие, своего рода догму. Мир много столетий, находясь под влиянием авторитета Аристотеля, считал, что Солнце вертится вокруг Земли.
                    давным-давно мир познаётся не зрением, а разумом

                    Разум ищет объяснения полученным наблюдениям. В смысле нашего обсуждения, разумом пытаются объяснить непонятные наблюдения, для этого придумывая недостающие детали.
                    А может быть все проще? Допустить, что на свете есть процессы протекающие со скоростями выше световых. Тогда многое станет на свои места.
                    Поясню свою мысль. В радиотехнике есть понятие спектра сигналов. В силу нелинейности, например, усилителя вместо одной гармоники сигнала на его входе на выходе образуется множество боковых гармоник наряду с основной. Боковые гармоники могут отстоять от нее на достаточно большом расстоянии, если смотреть по спектру.
                    Точно так же процессы, протекающие со сверхсветовыми скоростями могут порождать «гармоники» с разными скоростями, в том числе ниже скорости света. Человек наблюдает эти «гармоники», но не может найти их первопричину, поэтому разумом пытается додумать до объяснения, удовлетворяющего его разум. Как понимаете, далеко не гарантия, что такие догадки соответствуют реалиям.


                    1. Druu
                      03.10.2017 12:19

                      > А может быть все проще? Допустить, что на свете есть процессы протекающие со скоростями выше световых. Тогда многое станет на свои места.

                      Тогда появится куча неразрешимых парадоксов. Поверьте, если бы возможность сверхсветового движения чего-то бы там «ставила на свои места» — то это не то что стало бы ведущей теорией, СТО просто бы не появилась. Т.к. ее приняли исключительно потому, что других вариантов не осталось (а их весьма старательно искали).


                      1. AlexeyVPolunin
                        03.10.2017 12:31

                        Приведите хотя бы один возможный неразрешимый парадокс, для примера.
                        Кстати, Эйнштейн говорил (возможно, эти слова ему предписывают), что он сам себе представляется мальчиком, играющим в камушки на берегу непознанного океана Природы.


                        1. Druu
                          03.10.2017 17:19

                          > Приведите хотя бы один возможный неразрешимый парадокс, для примера.

                          Например, электро-магнитные явления нарушают принцип относительности Галилея, то есть ведут себя по-разному в разных ИСО и, т.о., в плоском пространстве-времени невозможны (парадоксальны) сами по себе.


                          1. AlexeyVPolunin
                            03.10.2017 18:47

                            А причем тут процессы, протекающие со сверхсветовыми скоростями?


                            1. Druu
                              04.10.2017 00:58

                              Невозможность сверхсветовых скоростей — это не свойство движущихся частиц, а свойство пространства, в котором они движутся. В пространстве, которое допускает сверхсветовые скорости, не может существовать электро-магнитное поле. Но электро-магнитное поле существует. Вот вам и парадокс. Именно этот парадокс и послужил вообще причиной появления СТО, к слову.


                              1. AlexeyVPolunin
                                04.10.2017 15:26

                                Пространство это наше восприятие окружающей нас материи.
                                Парадокса с электромагнитными волнами нет, есть только неверно выбранная точка отсчета. Внутри космического корабля, летящим со сверхсветовой скоростью, будут соблюдаться все физические законы для скоростей меньше скорости света. Для наблюдателя вне такого космического корабля этот корабль просто исчезнет, станет своего рода темной материей. В ее существовании вроде уже никто не сомневается, но обнаружить пока не может :)
                                Естественно, связь между наблюдателем в таком космическом корабле и наблюдателем на Земле с помощью электромагнитных волн станет невозможной.


                                1. Druu
                                  04.10.2017 15:56

                                  > Парадокса с электромагнитными волнами нет, есть только неверно выбранная точка отсчета.

                                  Конечно, есть. Вот у вас есть перрон, мимо него едет поезд (с ненулевой скоростью). Параллельно поезду летит фотон. Если вы рассчитаете скорость этого фотона при помощи уравнений Максвелла в ИСО перрона — она будет с, а если в ИСО поезда — то тоже с. С другой стороны, согласно галилеевскому принципу сложения скоростей — скорость относительно поезда должна быть меньше на x, где х — скорость поезда. Но с != c — x, получаем противоречие (парадокс). Либо неверны уравнения Максвелла, либо — преобразования Галилея, а поскольку преобразования Галилея сами выводятся из структуры пространства-времени — то значит и эта структура неверна. Либо одно — либо другое.

                                  Никаких космических кораблей и еще чего вы там себе навыдумывали не нужно. У нас просто есть два уравнения, которые дают разный результат — это и называется парадоксом. Какое-то из этих уравнений совершенно однозначно не является верным.


                          1. Tyusha Автор
                            03.10.2017 18:55

                            Чего?!


                            1. Druu
                              04.10.2017 00:58

                              Прочитайте историю создания СТО.


                    1. Lennonenko
                      03.10.2017 13:57

                      о боже, да хватит уже опровергать эйнштейна, есть кому этим заняться и без вас
                      если вы думаете, что как только теорфизик какой-нибудь получает в расчётах скорость, превышающую скорость света, он немедленно бросает всю работу и идёт каяться портретам эйнштейна, лоренца и максвелла, вы глубоко заблуждаетесь
                      всё учитывается, всё допускается, постоянно рождаются и испытываются новые гипотезы, рассматриваются и мнимые массы, и отрицательная плотность вакуума, и тахионы с тахионными полями, и супербрадионы
                      от скорости света зависит причинность событий, объекты, могущие передавать информацию и перемещающиеся быстрее света, будут причинность нарушать
                      эйнштейн много чего говорил и ещё больше ему приписывают или искажают, это не имеет отношения к дискуссии


                      1. AlexeyVPolunin
                        03.10.2017 14:31

                        Расслабьтесь :) Никто Эйнштейна не опровергает. Кстати, уважая этого ученого и других упомянутых ученых могли бы их фамилии писать с заглавной буквы.
                        Я умолкаю, ибо начинается «охота на ведьм» :) Всем хорошего настроения и доброго дня!
                        P.S. А упомянутый парадокс я так и не услышал :)


                    1. Tyusha Автор
                      03.10.2017 14:43

                      Вы не подозреваете, насколько вы правы. В квантовой теории поля учитываются формально сверхсветовые гармоники виртуальных частиц.


                      1. AlexeyVPolunin
                        03.10.2017 15:24

                        Вы не подозреваете, насколько вы правы.

                        Подозреваю :)


                      1. AlexeyVPolunin
                        03.10.2017 19:03

                        Кстати, а что мешает ситуацию развернуть на 180 град.? Формально посчитать, что существуют виртуальные процессы со скоростями выше скорости света и попробовать объяснить кое-что из квантовой физики иначе.
                        Сильно не наезжайте, далее пойдет импровизация, родившаяся только что. :)
                        Снова вернемся к радиотехнике. Есть синусоидальный сигнал определенной амплитуды. Есть усилитель, через который этот сигнал пропускается. Когда коэф.усиления невысокий, на выходе усилителя синусоида просто увеличивает свою амплитуду без искажений. Но если дать высокий коэф.усиления, то пики (максимумы) амплитуды синусоиды срезаться. Это очень упрощенно, на деле несколько не так, но для иллюстрации идеи, будем считать, что пики срезаются. На выходе получим что-то похожее на трапецеидальный меандр. С т.з. муравья, ползущего по синусоиде, начало и конец вершины трапеции меандра это перескок из одного состояния синусоиды в другое. Ничто не напоминает?


                        1. Tyusha Автор
                          03.10.2017 23:27

                          Честно — нет. :)


                          1. AlexeyVPolunin
                            04.10.2017 15:37

                            Боюсь подвергнутся осуждению, переходящего в анафему, но давно склоняюсь к мысли, что в природе все процессы имеют аналоговый, непрерывный характер. Квантование возникает, как в примере с синусоидой и муравьем, когда скорость процесса превышает скорость получения информации об этом процессе.


                            1. Druu
                              04.10.2017 15:59

                              Как в непрерывном случае разрешается ультрафиолетовая катастрофа?


                              1. AlexeyVPolunin
                                04.10.2017 16:14

                                В рамках физики досветовых скоростей никак.


                                1. Druu
                                  04.10.2017 16:19

                                  А в сверхсветовых как?


                                  1. AlexeyVPolunin
                                    04.10.2017 17:52

                                    На этот вопрос сможет ответить Эйнштейн-2. Я не гожусь на его роль :)


            1. IvanKor2017
              01.10.2017 20:42

              1. AlexeyVPolunin
                01.10.2017 20:45

                1. IvanKor2017
                  01.10.2017 21:05

                  Все источники бета-ИИ дают такой свет, даже в воздухе. Бета -излучение имеет сплошной энергетический спектр поэтому об эффекте Вавилова-Черенкова и речи не может быть. Что происходит в ускорителях отлично рассказано тем кто на них работал.


            1. sumanai
              01.10.2017 20:43

              Косвенно это подтверждает эффект Вавилова-Черенкова.

              Каким образом?



        1. vedenin1980
          01.10.2017 20:22
          +2

          А откуда слепые люди узнали бы о существовании света?

          А откуда мы узнали о радиоволнах? Или радиоктивном излучении, электрическом токе?


        1. sumanai
          01.10.2017 20:23

          Оттуда же откуда и мы, не видящие ни электромагнитное излучение, ни радиацию, ни нейтрино, знаем об их существовании.


          1. AlexeyVPolunin
            01.10.2017 20:28

            Электромагнитные излучения протекают со скоростью света. Поэтому их можно зафиксировать.


            1. sumanai
              01.10.2017 20:41

              Вы хотите сказать, что существует пятое взаимодействие со скоростью выше световой, и из-за этого мы его не замечаем?



            1. IvanKor2017
              01.10.2017 20:48

              1. AlexeyVPolunin
                01.10.2017 20:51

                Вы хотите сказать, что за открытие этого эффекта зря дали Нобелевскую премию по физике?


                1. IvanKor2017
                  01.10.2017 21:17

                  Было трудно но достали и Нобелевскую премию. Все остальное уже сказал. Что такое сплошной спектр бета-излучения погулите сами. Кстати, этим бета- излучение ИИ отличается от электронных ускорителей релятивистских электронов.


        1. Tyusha Автор
          02.10.2017 02:04

          del


        1. Lennonenko
          03.10.2017 01:26

          учитывая весь тот бардак, который происходит в зрительной коре и чуть глубже, люди так-то не особо-то и зрячие


        1. electrosnake
          03.10.2017 01:49

          Ну как, наблюдая, например, за поведением зрячих животных. При супрессии органа зрения у них менялось бы поведение.
          Другое дело, что от изобретения источника света (которым может служить банальная спичка, свеча, факел) до изобретения оптикоакустического преобразователя очень длинный путь, и вся наука развивалась бы по-другому.
          Но скорость света очень долго не считали максимальной, Митчелл изобрел черные звезды как тела с космической скоростью больше нее, но не было идеи ещё, что нельзя из мощной пушки выстрелить с поверхности такой звезды, чтобы улетело на бесконечность. Эта идея возникла, когда Эйнштейн сообразил, что скорость света во всех системах отсчета должна быть одинаковой (что приводило к разного рода «абсурдным» следствиям, вроде обсуждающихся в посте, но он оказался в силах их принять и обнаружить отсутствие в них истинных логических противоречий, как ранее Лобачевский с Гауссом и Бойяи в гиперболической геометрии; собственно, пространство скоростей в СТО эту геометрию и образует в определенном смысле).
          Что скорость распространения звука в разных системах отсчета разная (потому что среда «набигает»), обнаружили бы быстро, «звуковой эфир» нет проблемы поймать.


          1. AlexeyVPolunin
            03.10.2017 07:43

            Ну как, наблюдая, например, за поведением зрячих животных.

            Если под словом «наблюдая» вы понимаете «слушая», учитывая слепоту гипотетического человечества, то узнать, что в природе есть свет невозможно.
            Можно понять, как в примере с животными, что у них есть свой стиль поведения. В определенный промежуток времени они спят, в другой бодрствуют. Слепое человечество нашло бы свое, как им казалось бы, разумное объяснение. Скорее всего, они бы предположили о существовании некого физического процесса, влияющего на поведение животных. Но описать свет так же как это могут сделать зрячие люди, они бы не смогли.
            Кстати, на эту тему стоит расспросить офтальмологов, как понятие свет пытаются объяснить незрячим от рождения людям.
            Чтобы не дублировать посты, рекомендую прочитать этот мой комментарий


            1. michael_vostrikov
              03.10.2017 14:19

              то узнать, что в природе есть свет невозможно

              Почему это? Если вы не можете придумать такой эксперимент, это не значит, что он невозможен.


              Берем обезьяну и банан. Обезьяну фиксируем в кресле. Прикрепляем датчики положения головы. Бесшумно перемещаем банан из стороны в сторону вокруг обезьяны, проверяем, поворачивает ли она голову вслед за ним. Для чистоты эксперимента проводим его во время суток когда обезьяны проявляют активность и в условиях обитания обезьян. Обезьяна следит за бананом. Делаем вывод, что есть дополнительное взаимодействие, не связанное со звуком.


              Другой эксперимент. В теплый день приходим в место где тепло чувствуется сильнее. Помещаем над собой широкий крупный предмет — зонт или навес. Отмечаем уменьшение ощущения тепла. Делаем вывод, что есть излучение, не связанное со звуком, которое проникает в окружающей среде но не проникает сквозь предмет.


              1. AlexeyVPolunin
                03.10.2017 14:35

                Делаем вывод, что есть излучение, не связанное со звуком, которое проникает в окружающей среде но не проникает сквозь предмет.

                И почему слепое человечество посчитает это излучение светом?
                Может быть они его воспримут как радиоактивное излучение, от которого защищает зонтик из свинца.
                Последнюю фразу не воспринимайте серьезно. Это шутка.


                1. michael_vostrikov
                  03.10.2017 15:02

                  Они посчитают его излучением и будут изучать его свойства. Так же как изучают люди в нашей вселенной. Как они его назовут это дело десятое.
                  Радиоактивное излучение, как ни странно, это форма электромагнитного излучения, так же как тепло и свет.


                  1. AlexeyVPolunin
                    03.10.2017 15:30

                    Не сомневаюсь, что будут изучать и соглашусь, что не в названии дело. Только вы в первом своем посту смешали свет и тепло. Это немного разные вещи, хотя в случае, например, инфракрасного излучения они идут вместе. Для зрячего зонтик отсечет тепловое излучение и частично видимый свет, создав тень. Для слепого тени не будет вообще. Он заметит лишь изменение теплового потока. Об этом я написал здесь


                    1. michael_vostrikov
                      03.10.2017 22:26

                      И? В чем разница? В одном случае изменение сигнала с одних органов чувств, в другом с других. Существование излучения заметно в обоих случаях. Значит ваше утверждение о "невозможно" неверно.


                      И да, если вы в жаркий день подставите руку под солнечный свет и попробуете закрывать ладонью другой руки разные участки, то почувствуете тень даже с закрытыми глазами.


                      1. AlexeyVPolunin
                        03.10.2017 22:40

                        Вы говорите про тепловую тень. Это не световая тень.


                        1. michael_vostrikov
                          04.10.2017 10:35

                          А в чем разница-то?


                          И нет, закрываются именно солнечные лучи. Тепло появляется когда энергия из них переходит в поверхность, на которую они попадают. Если это были тепловые лучи, то чем выше тем должно было быть теплее. Вспомните Незнайку.


                          1. AlexeyVPolunin
                            04.10.2017 15:18

                            Разница в том, что Незнайка был зрячим.
                            Слепой от рождения никогда не узнает о существовании света, пока ему не расскажут. Солнечные лучи он будет считать тепловыми лучами, не более. У него будет тепловая тень под зонтиком. Цикличность день-ночь он будет воспринимать не как светло-темно, а как теплее-холоднее.


                            1. michael_vostrikov
                              04.10.2017 19:32

                              Причем здесь, зрячие они были или нет? От этого как-то должно меняться, что с увеличением высоты не становится теплее? Нет. Отсюда вывод, что тень от солнечных лучей это световая тень, а не тепловая, как вы ее назвали. Следовательно, вы неправы.


                              Еще раз. Неважно, кто что будет чувствовать. У обезьяны, поворачивающей голову за бананом, никакой тепловой тени нет. У растений на свету и не на свету будет разное развитие, независимо от одинаковой температуры. Радиоволны мы вообще никак не воспринимаем, но тем не менее обнаружили их существование. Отсюда вывод, наличие излучения можно обнаружить экспериментально. Следовательно, ваше высказывание неверно.


  1. AlexeyVPolunin
    01.10.2017 20:54
    +1

    Кстати, пример с инфракрасным излучением очень хорошо иллюстрирует мою мысль. Зрячий человек видит излучение и чувствует тепло. Как понимаем, разными органами чувств. Слепой только чувствует тепло. Но само-то инфракрасное излучение от этого не меняется. Меняется только наши представления о нем.


  1. Iron_69
    01.10.2017 21:23

    А вот интересно, выше было указано что «наша скорость относительно реликтового излучения составляет 627±22 км/с», а что было бы если бы мы смогли запустить корабль в ту сторону откуда движемся и разогнали бы его до тех же 627 км/с, т.е. добились бы того что бы скорость корабля относительно реликтового излучения стала бы нулевой. Что тогда? Время ускорится? На сколько? Вот мы знаем что при скоростях близких к световой время замедляется, а что на другом конце? При нулевой скорости какое будет течение времени? Будет стремиться к бесконечно большой?


    1. sumanai
      01.10.2017 21:31

      Будет стремиться к бесконечно большой?

      Нет.


  1. vedenin1980
    01.10.2017 21:31

    скорость корабля относительно реликтового излучения стала бы нулевой. Что тогда? Время ускорится? На сколько?

    Тут уже писал. Да, ускорится, но очень на крошечный процент. За каждую неделю на Земле часы на таком корабле будет спешить примерно на 1 секунду.

    При нулевой скорости какое будет течение времени? Будет стремиться к бесконечно большой?

    на крошечный процент (1,0000015) больше чем на Земле


  1. favik
    02.10.2017 02:05
    -1

    Господа! Я все понял! Какие то негодяи давным-давно в далекой-далекой галактике научились гонять со скоростями близким к скорости света и насоздовали лишнюю массу во вселенной, а земляне теперь мучаются с темной материей. А черные дыры — это нарушители скорости, засиделся у друзей, а тут жена звонит и домой требут срочно, а то развод. И все, дал по газам больше нормы, приблизился к скорости света и сиди теперь вечность, притягивай к себе все материю вместе с фотонами, и думай о своем поведении!


  1. Dessloch
    02.10.2017 10:52
    -2

    Надо понять что в аббревиатурах СТО и ОТО «Т» означает теория. Всего-лишь теория.


    1. sumanai
      02.10.2017 17:44

      Всем бы теориям иметь такие точные экспериментальные подтверждения, как теория относительности.


    1. Lennonenko
      03.10.2017 01:58

      а что в вашем понимании происходит с теориями, когда они уже больше, чем теории, и как их после этого называют?


    1. tundrawolf_kiba
      03.10.2017 18:58

      Всего-лишь теория.

      Вы наверное попутали с гипотезами. Гипотеза — это да, это «всего-лишь», пишется кстати без дефиса, вероятностное предположение. Теория же это уже сформированная и доказанная система идей или принципов, то с чем можно эффективно работать. Фактически — теория — это гипотеза, получившая подтверждение.


      1. Dessloch
        04.10.2017 07:11

        Что именно получило подтверждение? Что скорость света-предел? Или что если лететь со скоростью света то видимые объекты начинают менять свои размеры? А эфир всё-таки есть или его нет?


  1. kauri_39
    02.10.2017 21:11
    -2

    "Одновременно с ним с Альфы Центавра навстречу отправилась летающая тарелка инопланетян с той же скоростью."


    Тарелка, пожалуй, может лететь и с большей скоростью. Она же вращается и увлекает за собой пространство — заставляет его вращаться в том же направлении. Такой эффект, в частности, вращает орбиту Меркурия. То есть тарелка будет отклонять в сторону от себя все космические пылинки и микрометеориты, которые попадутся у неё на пути. Земной реактивный корабль такой защиты не имеет и вряд ли наберёт высокую скорость.


    Интересно, ваши студенты не сомневаются в точности ОТО, поскольку она не может верно описать вращение галактик и кластеров без добавки к ним тёмной материи? Тогда как частицы ТМ до сих пор не найдены, и космологическая LCDM-модель не во всём соответствует наблюдениям (проблема иерархии галактик).
    Потом, автор СТО и ОТО свято верил в максимальность скорости света для обмена информацией. Однако между запутанными частицами всё-таки существует "чудовищное дальнодействие", что подтвердила проверка неравенств Белла. То есть сами эти частицы могут, в отличии от нас, мгновенно обмениваться информацией, без которого их взаимодействие было бы невозможным.
    Или студенты теперь — народ прагматичный, романтизм первооткрывателей им не знаком?


    1. Tyusha Автор
      03.10.2017 02:44
      +1

      Меня удивляет как ваша неплохая осведомлённость в некоторых вопросах уживается с чудовищной ересью… Или это троллинг, тогда я не поняла…


      1. tundrawolf_kiba
        03.10.2017 19:03

        Это один из местных фриков. Человек без фундаментальных знаний пытается построить альтернативную версию вселенной. Отсюда и появляется и 5 измерение, и разумные фотоны…


      1. kauri_39
        03.10.2017 23:44

        Значит, студенты у вас смирные, подобной ересью не донимают. Однако это здесь и сейчас мои идеи так воспринимаются. Ваша молодёжь наверняка доживёт до новой физики, которая подтвердит их справедливость.
        Знание математики для физика обязательно. Но математика довозит его туда, куда его рулили его общие, философские представления о мире. И куда они дорулили? К проблеме космологической постоянной, к поискам незримой тёмной материи, к затянувшемуся созданию квантовой теории гравитации.
        Вас уже завтра могут спросить: почему когда теория Ньютона не смогла объяснить особенность вращения Меркурия её сменила теория Эйнштейна, а когда последняя не смогла объяснить особенность вращения галактик, то менять её на новую теорию не стали, а добавили к галактикам невидимой тёмной материи? Ведь и для объяснения вращения орбиты Меркурия по теории Ньютона можно было добавить невидимой в телескопы планеты Вулкан? Что ответите? Что у искателей планетного влияния на орбиту Меркурия было меньше фантазии и больше честности, чем у современных космологов?


        Какой вы сами видите путь к созданию работающей теории квантовой гравитации? Аналог электромагнитного притяжения здесь не катит, взаимно притягивающиеся тела не обмениваются квантами гравитационного поля. Почему бы не вернуться к гипотезе Ньютона о меньшем давлении эфира в телах по сравнению с его давлением вне тел? Называйте эту среду "энергетически плотным физическим вакуумом" — в соответствии с нынешними "догматами веры". Не хотите заморачиваться 5 измерением (в моей первой публикации), можно ограничиться постулированием снижения плотности физвакуума материей. Подтверждением чего является снижение в поле гравитации частоты фотонов, задающих темп хода атомных часов, эффект Казимира, и недавно открытое притяжение атомов к излучающим тепловые фотоны телам. Даже тягу EmDrive объяснить можно.
        И быстрое вращение галактик и кластеров объясняется без ТМ! И без неё первичная горячая материя, непослушная гравитации, быстро соберётся в протогалактические туманности. Потому что пространство с большей концентрацией материи в любом её виде имеет меньшую плотность и скорость расширения, и поэтому его будет сжимать внешнее для него пространство с меньшей концентрацией материи. Вот почему первичные малые флуктуации материи привели к её наблюдаемому ячеистому распределению во Вселенной.


        Разумные фотоны с их мгновенной связью — это дополнительная опция в моей теории, для самых продвинутых. Тут уже требуется вывод общего закона эволюции (последняя публикация) и соответствующий вариант мультиверса с 5 измерением. Зато проверку делать уже не надо — Белл и компания постарались.


        Попробуйте расшевелить студентов этими идеями. Или намекните на финансовый интерес — от гонорара за выражение идей в фантастической книге до Нобелевки за их выражение в математике (будет лучше, чем у Милгрома и Верлинде). А то что они подумают про нашу родину, работая за рубежом, — и общество у нас отсталое и физика прошлого века.


  1. photino
    03.10.2017 00:25
    +1

    Не уверен, что мой комментарий пройдет модерацию быстрее, чем за полгода (как было с предыдущим), но все же попробую :-)

    Автор несомненно разбирается в СТО и ОТО, но все же дифференцировать формулу (1) по времени некорректно. Она верна только в инерциальной системе отсчета, то есть при постоянной скорости v. Если перейти в неинерциальную систему отсчета и попробовать в рамках ОТО получить формулу, аналогичную формуле (1) (понятно, что с определением расстояния там все будет не так тривиально, как в СТО), то по идее в ней будет еще какая-либо добавка, пропорциональная ускорению. Так что вместо (5) может получиться что-то совсем другое, возможно кардинально другое. Честно говоря, проверить все это просто нет времени, но из общих соображений это должно быть так.

    Что касается сверхсветовых скоростей при расширении Вселенной, то это немного другое — расширение самого пространства некорректно сравнивать с движением объекта в пространстве.


    1. Tyusha Автор
      03.10.2017 00:46

      Пожалуй, первый комментарий по делу.

      дифференцировать формулу (1) по времени некорректно. Она верна только в инерциальной системе отсчета, то есть при постоянной скорости v.


      Вы хотите сказать, что надо провести геодезическую и измерить её длину? Почти готова согласиться, но что-то поздно, думать лень.

      Если я правильно понимаю, вы хотите сказать, что в 4-пространстве линия (конечно пространственно подобная), по которой прикладывается линейка до Альфы, изогнута. Т.е. если рассмотреть сопутствующую СК в какой-то момент ускорения, то эта линия начнётся вдоль оси x', а ближе к Альфа загнётся вверх (касательный вектор получит t-составляющую)?

      Вопрос не простой, тут думать придётся.

      А насчёт ОТО я не согласна, что её все сюда тянут. Да приходится рассматривать искривлённое пространство с случае ускорения или вращения, но для этого не нужен ни один из постулатов ОТО! Да-да. Поэтому настаиваю, что это СТО, хоть и с аппаратом привычным ОТО. Но лишь малой его частью.


      1. Tyusha Автор
        03.10.2017 00:50

        Добавлю, что для малых отрезков формула (1), разумеется, верна вообще в любой СО.


      1. photino
        03.10.2017 02:41
        +2

        Сейчас попробую пояснить более детально. Я понимаю, что Вы имеете в виду, говоря о том, что «приходится рассматривать искривлённое пространство с случае ускорения или вращения, но для этого не нужен ни один из постулатов ОТО», и в этом смысле с Вами согласен. Для получения метрики в ускоренной системе координат действительно не нужны уравнения Эйнштейна и т.п. Когда я говорил про ОТО, я имел в виду вопрос измерения расстояний и промежутков времени. Сейчас специально залез в «Теорию поля» Л/Л, чтобы освежить в памяти, там есть параграф «Расстояния и промежутки времени» (глава X, параграф 84 в издании 1988 г.) Как Вы знаете, бесконечно малые расстояния можно измерять, пуская свет по геодезической до нужной точки и обратно. А вот проинтегрировать для получения конечного расстояния в общем случае не получится — так как эффективная метрика может зависеть от времени, результат будет зависеть от мировой линии, по которой ведется интегрирование. Интегрировать можно, когда метрика не зависит от времени. Собственно, у Л/Л написано, что «Таким образом, в общей теории относительности теряет, вообще говоря, смысл понятие об определенном расстоянии между телами, остающееся в силе лишь в бесконечно малом.»

        Теоретически, можно пустить свет, который полетит по геодезической до объекта назначения (Альфа), дождаться возвращения и посчитать расстояние. Но в этой неинерциальной системе координат Альфа движется к нам, так что не очень понятно, что в итоге будет намерено таким способом (ну то есть если бы мы измеряли расстояние до приближающейся машины, отправляя к ней мотоциклиста, а по его возвращении умножая его скорость на время в пути — получается не то, что нам нужно в реальности, особенно если мотоциклист не сильно быстрее автомобиля).

        Возможно, в случае равноускоренной системы отсчета есть какие-то особенности, которые упрощают задачу, но с ходу я их не вижу. Как загнется (или не загнется) линия я, честно говоря, тоже с ходу не знаю. По хорошему, нужно все честно посчитать и посмотреть, там все довольно просто, ну или найти — практически уверен, что в каком-либо из учебников это есть (если не ошибаюсь, парадокс близнецов с учетом ускорения рассматривался в учебнике Макса Борна «Эйнштейновская теория относительности», можно там посмотреть). Допускаю, что может получиться следующее — даже если Вы посчитаете как-то расстояние (с учетом проблем, указанных выше), не получится взять какое угодно большое расстояние, чтобы получить какую угодно большую скорость — метрику неинерциальной системы отсчета можно будет использовать только при ограниченных временах/расстояниях (как в случае вращения, см. параграф 89 в Л/Л). Хотя скорость, определенная как полное расстояние по замкнутому контуру, деленное на полное время, затраченное светом на путь, действительно может оказаться больше скорости света, как во вращающейся системе отсчета.

        Так что да, вопрос не такой уж и однозначный. Но Вы молодец, что пытаетесь разобраться и продемонстрировать на простых примерах пределы применимости теории :-)


        1. Tyusha Автор
          03.10.2017 03:00
          +1

          Снимаю шляпу. Теперь согласна однозначно. Действительно надо честно интегрировать, нельзя брать L «одним куском». Только тс-с-с, никому не говорите. Кроме вас никто не заметил. :)

          Но эффект сверхсветовой скорости, несмотря на косячность определения скорости есть, со скидкой на не точность самого определения. Это видно на примере задачи. Ракета изменила скорость (как-то ускорилась), а тарелка вдруг оказывается проскочила большое расстояние, которое и до и после ускорения мы измеряем вполнн корректно — в инерциальных СО. Значит где-то в промежутке имела место сверхсветовая скорость, скажем лучше «быстрота» приближения, что обойти строгость определений.


          1. Tyusha Автор
            03.10.2017 03:07

            Насчёт сколь угодно большого расстояния, надо заметить, что там ещё и горизонт событий сзади по курсу возникает.


          1. photino
            03.10.2017 10:56
            +1

            Только тс-с-с, никому не говорите.

            ОК, без проблем :-)

            Я тут с утра еще немного подумал про Ваше наблюдение, в общем, его можно объяснить если не проще, то, на мой взгляд, немного более последовательно и корректно. Надеюсь, Вы не обидитесь на небольшие правки :-) Будет немного LaTeX'а и много занудства. Вместо тарелки для простоты взял Альфу Центавра.

            Сначала посмотрим, сколько времени займет путешествие со скоростью v от Земли до Альфы в лабораторной системе отсчета (связанной с Землей и Альфой). Получим t=L/v. Теперь представим, что корабль стартует из точки на расстоянии l от Земли со стороны, противоположной Альфе, и разгоняется в течение времени T с постоянным ускорением A (l, A и T измеряются в лабораторной системе отсчета) в сторону Альфы таким образом, чтобы в момент пролета мимо Земли достичь скорости v (опять же, измеряется в лабораторной системе отсчета) и дальше лететь к Альфе с этой постоянной скоростью. В момент встречи с Землей в системе отсчета корабля расстояние до Альфы равно L\sqrt{1-v^{2}/c^{2}} (останавливаться на том, как формально можно измерить это расстояние в системе отсчета корабля, я не буду — Вы и так это знаете :-)). В этой системе отсчета Альфа летит навстречу кораблю со скоростью v, так что Альфа долетит до корабля за время (по часам корабля) L\sqrt{1-v^{2}/c^{2}}/v=t\sqrt{1-v^{2}/c^{2}}. Так и должно быть — по сравнению с часами на Земле и Альфе, часы на корабле идут медленнее.

            Как теперь получить сверхсветовые скорости? Для этого нужно сделать некую вещь, которая, по сути, является сравнением красного с круглым. Мы вычтем из расстояния от корабля до Альфы после окончания ускорения L\sqrt{1-v^{2}/c^{2}} расстояние от корабля до Альфы до начала ускорения l+L. Эти расстояния измерены в разных системах отсчета, поэтому такая операция, конечно же, некорректна. Но для команды корабля что расстояние до старта, что расстояние на момент окончания ускорения измеряются в одних и тех же единицах, например, в парсеках или миллиметрах, как им больше нравится, так что почему бы и не вычесть одно из другого :-). В предположении, что v/c<<1, получим -(l+Lv^{2}/(2c^{2})). Для того, чтобы посчитать среднюю скорость приближения Альфы с точки зрения команды корабля, нужно посчитать время, затраченное на ускорение. Собственное время корабля, затраченное на ускорение, вычисляется как \tau=\int_{0}^{T}\sqrt{1-A^{2}t^{2}/c^{2}}dt (можно сказать, что вот здесь было совсем немного ОТО), это время команда корабля увидит на своих часах по окончании разгона. При AT/c<<1 оно примерно равно T. В результате с учетом того, что l=AT^{2}/2 и v=AT, для средней «скорости» приближения Альфы получим V=-(v+AvL/c^{2})/2. Забавно, но с точностью до 1/2 это формула (5) в соответствующем приближении (хотя, если задуматься, как раз такая комбинация во втором слагаемом подходит по соображениям размерности, если предположить линейность по ускорению).

            Еще раз обращаю внимание, что процедура сравнения расстояний, измеренных в разных системах отсчета, как формально, так и фактически является некорректной. Более того, если бы команда корабля с помощью ОТО попыталась измерить все расстояния в своей системе отсчета, которая в момент разгона была неинерциальной (и получить формулу, которую действительно можно дифференцировать по собственному времени корабля и вычислять скорость просто по определению), то ничего бы не получилось — конечное (не бесконечно малое) расстояние в такой системе, судя по всему, вообще нельзя корректно определить (лучше бы это проверить, но я сомневаюсь в положительном ответе). Соответственно, и скорость приближения Альфы нормально посчитать не получится, а рассчитанная выше «скорость» не имеет отношения к какой-либо реально измеренной скорости (ну Вы как раз писали про «косячность определения скорости» :-)). Но если абстрагироваться от этих деталей и посмотреть на ситуацию с точки зрения команды корабля, которая вычисляет скорость таким вот специфическим образом (ну вот как умеет), то за время разгона корабля Альфа действительно вроде как быстро приблизилась к кораблю (а при определенных условиях, со сверхсветовой средней скоростью).

            Для не сильно разбирающихся в теме добавлю, что когда говорится о скорости света в искривленном пространстве-времени, подразумевается скорость света в локально инерциальной системе отсчета, которая, по сути, является плоским пространством в малой окрестности выбранной точки. А иначе действительно можно наткнуться на разные кажущиеся парадоксы. Как я понимаю, автор как раз и хотела продемонстрировать один из таких кажущихся парадоксов, за что ей спасибо — эффект действительно забавный и довольно занятный.


            1. photino
              03.10.2017 11:24

              Еще раз обращаю внимание, что процедура сравнения расстояний, измеренных в разных системах отсчета, как формально, так и фактически является некорректной.

              Конечно, здесь я имею в виду сравнение именно в контексте вычисления скорости, а не сравнение как таковое.


            1. Tyusha Автор
              03.10.2017 14:02

              Всё так, проверила. Только осталось усреднить v по отрезку разгона, т.е. заменить v \rightarrow v/2 и… та-дам… (5).


              А насчёт круглого с красным. Может оно и не корректно, но "дальномер" на борту ракеты ведь что-то должен показывать?


              Я понимаю, что дальномер в неинерциальной системе откажет, т.к. непонятно как запускать "мотоциклиста". Опять-таки умозрительно тут что-либо сказать сложно. Или не откажет? Ведь равноускоренная метрика сама по себе стационарна, там на первый взгляд не должно быть пробле параграфа 84. (Признаюсь, пока руки не дотянулись со вчера до Л/Л). Другое дело, если рассматривать метрику, связанную с ракетой, которая то движется равномерно, то решает вдруг ускориться.


              1. Tyusha Автор
                03.10.2017 14:12

                Ошиблась, конечно надо сделать обратную замену v на 2v, но вы и так поняли.

                Вообще я опасалась, когда писала пост, что в комментариях основное бурление будет вокруг процедуры измерения расстояния. Опасалась, что это очень непростой момент, и дискуссия завязнет в нём. В то время как на основную канву и суть поста не влияет.


                1. photino
                  03.10.2017 20:38
                  +1

                  Только осталось усреднить v по отрезку разгона, т.е. заменить v \rightarrow v/2 и… та-дам… (5).

                  Дополнительно усреднять уже не нужно — усреднение получается автоматически. Если возьмете L=0, тогда получится v/2 — средняя скорость движения с постоянным ускорением на отрезке l (старт-Земля), как и должно получиться.

                  Или не откажет? Ведь равноускоренная метрика сама по себе стационарна, там на первый взгляд не должно быть пробле параграфа 84.

                  Тут вопрос, как определить ускорение. Если как попроще, то нормально ничего не сделать. А если посложнее, то можно попробовать что-то сделать, хотя бы частично. Я вспомнил, где это было — в «Гравитации» Мизнера, Торна, Уилера, том 1, есть глава 6 под названием «Ускоренные наблюдатели». Так вот, можно рассматривать гиперболическое движение, там в лабораторной системе отсчета не просто v=at, где a постоянное, а закон движения посложнее, зато постоянен квадрат 4-ускорения. Там метрика ускоренной таким образом системы координат не зависит от собственного времени, хотя тоже имеет особенности. Посмотрите, может что и получится. В этой главе есть и про радиолокационный дальномер.

                  Другое дело, если рассматривать метрику, связанную с ракетой, которая то движется равномерно, то решает вдруг ускориться.

                  С этим все плохо и в случае гиперболического движения. У Мизнера, Торна, Уилера (в этой же главе, параграф «Ограничения на размер ускоренной системы отсчета») как раз про это написано — когда наблюдатель сначала движется с постоянной скоростью, потом ускоряется, потом опять движется с постоянной скоростью. В общем, во всем пространстве-времени такую систему координат не ввести, только в некой области. В общем, посмотрите сами.

                  Может оно и не корректно, но «дальномер» на борту ракеты ведь что-то должен показывать?

                  Показывать он может, но вопрос, насколько эти показания что-то реально отражают. Все же физическая скорость объекта — это то, что можно непосредственно измерить с помощью линеек и часов в локально инерциальной системе координат наблюдателя. И эта величина не будет больше скорости света. Конечно, можно вводить определения скорости для объектов, находящихся на расстоянии, с использованием как-то высчитанных расстояний до этих объектов, но если система отсчета не инерциальная, то все это от лукавого.

                  Кстати, при таких рассуждениях команда корабля может подумать, что это не Альфа к ним резко приблизилась со сверхсветовой скоростью, а они приблизились к ней с этой сверхсветовой скоростью :-) Собственно, вот и проблема корректности определения скорости относительно удаленного наблюдателя в общем случае.


                1. photino
                  03.10.2017 21:27
                  +1

                  Все же глянул для чисто гиперболического движения. В системе отсчета корабля, в которой нулевая координата — X^{0}, а координата по направлению к Альфе — X^{1}, можно посчитать производную от координаты Альфы в этой системе отсчета по координате X^{0}. Предположив, что в момент, когда в лабораторной системе отсчета скорость корабля равна 0, расстояние между кораблем и Альфой равно L, получится -(gL+1)th(gX^{0})/ch(gX^{0}) (скорость света положим равной 1). Здесь g — ускорение корабля в сопутствующей кораблю инерциальной системе отсчета в любой момент времени. При большом L при определенных X^{0} эта величина может быть по модулю больше 1.

                  Но финт в том, что это производная посчитанной нами координаты Альфы по собственному времени корабля. Собственное время Альфы в этой системе координат будет совсем другое, так что для правильно определенной скорости Альфы получим -th(gX^{0}), что по модулю меньше 1.


                  1. Tyusha Автор
                    03.10.2017 23:41

                    На досуге ещё покапаюсь.

                    По идее, тем кто на ракете нет дела, что там с собственным временем Альфы. Наверное надо с самого начала брать другие координаты, наверное риндлеровские. Но это уже было бы не для Geektimes.

                    А вообще надо провести занятные численные расчёты равноускоренного движения. Там экспонента, а значит жди чудес. К тому же «наброс» про теорию относительности хорошо зашёл здесь.

                    Огромное спасибо за комментарии, мне это было и полезно, и приятно.


                    1. photino
                      04.10.2017 00:20

                      По идее, тем кто на ракете нет дела, что там с собственным временем Альфы.

                      Конечно им нет дела, но тут вопрос просто о корректном определении скорости. Ну если я поделю расстояние от Земли до Солнца на время, которое затрачиваю на поход из комнаты на кухню, то получу величину, по размерности соответствующую скорости, причем эта величина будет больше скорости света. Но она, понятное дело, ни о чем.

                      наверное риндлеровские.

                      Точно, это координаты Риндлера! Никак не мог вспомнить, как они называются, а в Мизнере, Торне, Уилере с ходу не увидел.

                      Вам тоже спасибо! Я уже довольно давно работаю в другой тематике, так что многие вещи начал забывать, а тут выпал повод вспомнить и немного размяться :-) Удачи!


  1. mckokos
    03.10.2017 00:25
    -2

    что делать с вполне законной анигиляцией материи при приближении к скорости света?:)


    1. VenomBlood
      03.10.2017 04:55

      Что?


      1. Tyusha Автор
        03.10.2017 10:28

        Неужели это тот единственный комментарий, который вы оставили к этому посту? :) Познова-то подтянулись, думала раньше появитесь.


        1. VenomBlood
          04.10.2017 00:59

          Да что-то скучно стало одно и то же писать, из поста в пост то эфирщик приходит, то «я, конечно, бухгалтер, но в школе у меня была физика, и поэтому: ....», так что изредка только на что-нибудь странное отвечаю.


  1. Hemeg
    03.10.2017 11:36
    -1

    Но эффект сверхсветовой скорости, несмотря на косячность определения скорости есть, со скидкой на не точность самого определения. Это видно на примере задачи.


    И все таки Вы не шутите. :(
    Вот если заменить «эффект», на «иллюзия», то нормально.
    Поскольку СТО о наблюдении, будем этими категориям и оперировать.
    Вот мы ускорились, скорость увеличилась, это легко замерить по «доплеру», но по доплеру вы никогда, не будете лететь быстрее света. Это очевидно.
    А как измерить расстояние? Ну допустим, наша цель двойная звезда, можно замерить параллакс. И, о чудо, расстояние сократилось! То есть мы прилетим быстрее.
    Никакого наблюдаемого движения быстрее света!
    Эффект сверхсветовой скорости только в голове, если делить расстояние из прошлого измерения на время прибытия из текущего.

    ЗЫ Все таки вы нас тролите, признайтесь.


  1. ozonar
    03.10.2017 12:43

    Объясните, пожалуйста, на примере задачи:

    Есть Земля и Альфа, расстояние между ними — 1 световой год. Корабль полетел от Земли к Альфе со скоростью с/2, будем считать что ускорение и замедление — мгновенны.

    1. За какое время долетит корабль с точки зрения Альфы?
    2. За какое время долетит корабль с точки зрения корабля?
    3. За какое время долетит корабль с точки зрения Земли?


    1. Tyusha Автор
      03.10.2017 14:17

      1. 2 года
      2. 1,73 года
      3. 2 года


  1. Dessloch
    03.10.2017 15:47
    -1

    Вот интересная статья на эту тему.
    www.globalistika.ru/biblio/velosity_gravitation.htm


    1. Tyusha Автор
      03.10.2017 21:14

      Очередная бредуха. Пробежалась. Лишь некотррые комментарии:

      1. Обмен гравитонами может притягивать. Виртуальные частицы, а именно такими являются все гравитоны стационарного поля, передают отрицательный импульс, т.е. ведут к притяжения. Впрочем превижу ответ, что виртуальных частиц нет. Тут я уже пасую.

      2. Гегель и примкнувший к нему Энгельс — это с моей точки зрения фрики. В метафизике у них полный фейл по всем пунктам, какие бы они ни пытались затронуть. Последующий век над ними не раз посмеялся. Так что их мнение в этом вопросе вообще ничто. Диалектика — это… я культурно промолчу, т.к. её при совке настолько сбили всем в головы, что я ненароком ещё обижу старшее поколение. Скажу культурно: к научному методу она не относится никак. А в статье по ссылке аргументы: Гегель сказал то, Гегель сказал сё.

      3. И просто уровень автора в физических вопросах: не понимаю, поэтому это не так.


  1. Dessloch
    03.10.2017 17:03

    Мне тут молча наставили минусов, но как человек имеющий своё мнение и готовый за него драться, чувствую что обязан высказаться для будущих поколений, чтобы потомки не думали что в этот период времени абсолютно все уверовали в непогрешимость ТО.
    Эйнштейн-распиаренная заурядность и троечник, бывший клерк патентного бюро. Успехами своими обязан в чём-то бывшей жене, талантливой сербке Милеве Марич, Лоренцу и Пуанкаре, у которых «позаимствовал» часть идей. Долгое время Эйнштейну не удавалось получить Нобелевскую премию. Но потом его всё-таки пропихнули и дали нобелевку за исследование фотоэффекта. Открыл фотоэффект Столетов, а премию получил Эйнштейн.


    1. Tyusha Автор
      03.10.2017 19:16

      Я правильно понимаю вас: Эйнштейн — очень плохой человек, поэтому теория относительности неверна?


      1. Dessloch
        04.10.2017 05:10
        -1

        Нет, всё не так. Эйнштейн был заурядным троечником и клерком патентного бюро. Потом выдал свою ТО, что уже заставляет относиться к ней скептически.
        Что такое постулат в переводе на русский язык? Это ничем не доказуемая чушь, которая принимается как 100% факт. Обычный приём манипуляции сознанием. Вместо русского слова «враньё» используем умное иностранное слово «постулат». И если кто-то обнаруживает процессы, распространяющиеся со скоростью большей скорости света делаем удивлённое лицо и гневно вопрошаем: «Как?? Ваши результаты противоречат СТО и постулату постоянства скорости света! Этого просто не может быть!» Ну конечно, вся вселенная должна внимать бывшему клерку патентного бюро и вести себя в соответствии с его постулатами и теориями. Раньше сжигали на костре, сейчас можно затравить, лишить званий и учёных степеней, заминусовать на форуме.
        Нет тут никакой науки. Правильно Перельман высказался про шнобелевский комитет.


        1. photino
          04.10.2017 12:48

          Эйнштейн был заурядным троечником

          Ну вы погуглите его аттестат и посмотрите оценки по естественным наукам. Удивитесь.
          сейчас можно… заминусовать на форуме.

          Да, действительно трагедия.


        1. Druu
          04.10.2017 14:22

          > И если кто-то обнаруживает процессы, распространяющиеся со скоростью большей скорости света делаем удивлённое лицо и гневно вопрошаем: «Как?? Ваши результаты противоречат СТО и постулату постоянства скорости света! Этого просто не может быть!»

          «Постулатом» постоянство скорости света является лишь в СТО. А в электро-динамике — это теорема, причем теорема простая (на втором/третьем курсе математики/физики это все проходят) и известна была задолго до появления СТО.


    1. photino
      03.10.2017 20:56

      Не сомневаюсь, что потомки будут обшаривать весь интернет в поисках Ваших посланий будущему поколению.

      А Столетов умер еще до начала присуждения Нобелевских премий, поэтому, к сожалению, никак не мог ее получить.


  1. ZloyDooh
    03.10.2017 19:00

    Я не физик. Заинтересовал заголовок — прочитал пост. Комментарии не читал, поэтому, возможно, повторю чьи-то слова. Расстояние не зависит ни от скорости, ни от ускорения и не может становиться меньше )) А ускорение не может расти бесконечно. Поэтому, никакого уменьшения расстояния не произойдет. L так и будет равно L, с какой бы скоростью (досветовой) вы не двигались. Поэтому и световой год равен расстоянию, которое свет проходит за один год СО СКОРОСТЬЮ СВЕТА )) То есть, все, что написано, чушь, извините.


    1. Tyusha Автор
      03.10.2017 19:06

      Ваши тезисы напрямую противоречат теории относительности.


      1. DRIVER_71
        03.10.2017 23:46

        Мой интеллект 80IQ. Да, мне не понять многова. Но ещё больше я не понимаю ваших споров. Спор о предмете (физике) требующих больших практических навыков между двумя книжными червями… средней квалификации. Да кто сказал вам что вселенная устроена так как вы её себе представляете(или видите). Хомо-сапиенсу это не нужно, да и вредно. Нельзя научить собаку читать. Ей это не под силу. Человечеству никогда не пробиться сквозь скорлупу бытия. Это задачка для существ с другой структурой мозга. Где культура передаётся на генном уровне а интеллект от рождения несколько десятков тысяч. — «сидели б вы в траве покоса и во все дрязги не совали носа» (Гёте, мефистофель). Возможно, когда нибудь будет создан интеллект кремний-галлий-оптико-органический со сроком жизни пару десятков тысяч лет. Но всё на этом и закончится. Новые формы жизни не потерпят присутствия посторонних на их территории. Так что умники — похоронят человечество в очередной раз( кстати уже четвёртый наверное)


        1. ZloyDooh
          04.10.2017 01:12

          Я не спорю, а указываю девушке на очевидные факты.


      1. ZloyDooh
        04.10.2017 00:55

        Что конкретно противоречит и чему? ))


        1. Tyusha Автор
          04.10.2017 01:40

          Ваше утверждение, что расстояние не зависит от скорости, противоречит преобразованиям Лоренца в специальной теории относительности.


          1. ZloyDooh
            04.10.2017 01:43

            )) Очень смешно. Доказательства? Это теория Лоренца не противоречит СТО до определенных пределов ))


            1. Tyusha Автор
              04.10.2017 01:59

              Представьте себе, мне тоже смешно.


          1. ZloyDooh
            04.10.2017 01:49

            Вы можете, конечно, подтвердить это многочисленными экспериментальными данными с использованием сверхсветовых скоростей ))


      1. ZloyDooh
        04.10.2017 01:02

        Не поленитесь рассчитать скорость объекта, который начал двигаться со скоростью с/2 и с ускорением G (т.е. 9,8 м/c2) через год )) И не морочьте людям головы


        1. Tyusha Автор
          04.10.2017 01:25

          Это скорость будет практически равна 1с. Что не так?

          Поясню. Ускорение g поддерживается двигателями ракеты в собственной системе отсчёта. Именно такое движение и называется в СТО равноускоренным. В неподвижной системе это будет выглядеть, что ускорение асимптотически падает, скорость при этом стремится к скорости света, но разумеется, никогда не превройдёт её.


          1. ZloyDooh
            04.10.2017 01:38

            Ну уж нет, за год с нуля с таким ускорением скорость достигнет скорости света. А в вашем понимании она и дальше расти будет в системе отсчета ракеты? )) И двигаться на расстоянии L ракета тоже будет в своей системе отсчета? А для всех остальных она пройдет расстояние L за четыре световых года? )) Вам бы Центробанком рулить в его системе отсчета! ))


            1. Tyusha Автор
              04.10.2017 01:54

              Давайте начистоту. Вы не знаете и не понимаете теорию относительности. Без этих базовых знаний вы не сможете понять мои аргументы. Тем более вы не спрашиваете, а берётесь утверждать сами.
              Поверьте, это выглядит смешно.

              Пересказывать учебник здесь не место, и желания у меня нет. Исходя из этого, советую принимать мои ответы на веру. Я вас не обману. Если согласны — продолжаем, если нет — не вижу смысла.


              1. ZloyDooh
                04.10.2017 10:27

                Таки представите нам ссылку на результаты экспериментов, подтверждающих сокращение времени в космологических масштабах при скоростях, близких к скорости света.
                Требую разъяснения базового понятия «световой год», а то, возможно, что я о юридическом понятии толкую, а вы о физическом, а на деле, как говорится, это две большие разницы…


                1. Tyusha Автор
                  04.10.2017 11:26

                  1. Пример в вашем смартфоне — работающая GPS, выше в комментариях это обсуждали. Мне кажется, на этом можно закончить с примерами.

                  2. В космогическом масштабе пример — красное смещение. И вообще эффект Доплера.

                  3. Световой год — расстояние, которое проходит свет за год.


                  1. ZloyDooh
                    04.10.2017 12:22

                    А с чего вы взяли, что двигаясь со скоростью, менее световой, объект пройдет расстояние, проходимое светом за год, быстрее?

                    Эффект Доплера — всего лишь увеличение длины волны вследствии удаления источника излучения от наблюдателя или наблюдателя от источника излучения. Расстояние при этом сокращается только в случае сближения при наличии собственной скорости. В случае неподвижности обоих (источника и наблюдателя) никакого эффекта Доплера наблюдаться не будет. По условию задачи нужно узнать расстояние до Альфы и до тарелки В МОМЕНТ СТАРТА, так вот — оно будет равно 4 св. года и одинаково и для Альфы и для тарелки. )) Поскольку в момент старта это будет являться статичной системой (так в условии написано). Расстояние начнет сокращаться ПОСЛЕ старта на величину с учетом скорости движения ракеты и тарелки, вектор которой будет направлен к точке встречи и встретятся они через 4 года (с учетом скорости сближения), а полный путь каждый из них пройдет за 8 лет. Не согласны — повторите курс математики за 2 класс.
                    Про GPS читать не стал — поскольку не зная алгоритм работы (производимых вычислений) говорить об этом полагаю некорректным.


                    1. Tyusha Автор
                      04.10.2017 12:47

                      А с чего вы взяли, что двигаясь со скоростью, менее световой, объект пройдет расстояние, проходимое светом за год, быстрее?
                      .

                      Если вы о задаче, то потому, что в движущейся СО ракеты, расстояние уже будет меньше 4 св.лет из-за Лоренцо сокращения длины.

                      Если у вас есть претензии к решению, вы можете указать на ошибки в вычислениях конкретно, например по порядковому номеру формулы.

                      А эффект Доплера есть обычный (скажем в воздухе) и релятивсткий. Результаты там разные, т.к. в релятивиский происходит не только за счёт движения источника, то замедления времени в нём, что и наблюдается.


                      1. ZloyDooh
                        04.10.2017 13:17

                        Дело в том, что ошибки можно найти в принципиально ПРАВИЛЬНЫХ вычислениях, а когда Вы применяете неправильный подход к решению задачи — то получите в принципе неправильный ответ.
                        Как всем известно, расстояние есть произведение скорости на время. Двигаясь со скоростью с/2 за 4 года вы пройдете расстояние в 2 световых года (очевидно же). Ну и далее, в соответствии с законами элементарной математики ))
                        В приведенной формуле (1) скорость V ограничена скоростью света (большая, насколько мне известно, не наблюдалась, но Вы можете меня поправить).
                        Световой год — расстояние которое свет проходит за один год времени стороннего наблюдателя (а не год по времени двигающегося со световой скоростью объекта).
                        Чтобы доказать, что течение времени для стороннего наблюдателя и для движущегося объекта происходит по разному, нужно взять тяжелый элемент с известным периодом полураспада, разогнать его до световой скорости и посмотреть, что будет с этим элементом через контрольный период времени.
                        Но нужно помнить, что воздействие энергии на тяжелый элемент может изменить его энергетическое состояние и увеличить его период полураспада.
                        Может, Вы сможете разрушить мою логику и она неверна?
                        А про Лоренцо сокращение длины Вы расскажите Российским Железным Дорогам, они ухватяться за эту идею и, пожалуй, освоят два триллиона бюджетных рублей. Безуспешно, думаю. ))


                        1. photino
                          04.10.2017 13:38

                          Чтобы доказать, что течение времени для стороннего наблюдателя и для движущегося объекта происходит по разному, нужно взять тяжелый элемент с известным периодом полураспада, разогнать его до световой скорости и посмотреть, что будет с этим элементом через контрольный период времени.

                          Этот эффект давно наблюдается в природе — существенное увеличение времени жизни (с точки зрения наблюдателя на земле) мюонов в космических лучах.


                          1. ZloyDooh
                            04.10.2017 13:49

                            Что, безусловно, может быть связано с тем, что в космосе (вакууме) мюоны теряют энергию значительно медленнее ))


                            1. photino
                              04.10.2017 14:06

                              Не может


            1. Druu
              04.10.2017 01:54

              > А в вашем понимании она и дальше расти будет в системе отсчета ракеты?

              Скорость ракеты в СО ракеты, очевидно, всегда равна нулю.


              1. ZloyDooh
                04.10.2017 10:25

                Это был сарказм, кто не понял ))


      1. ZloyDooh
        04.10.2017 01:17

        И еще: объясните мне, что произойдет с частицей, состоящей из элементов с разными массами (протоны, нейтроны и другие, более мелкие), двигающейся со скоростью света. )) Со дна своего юридического образования я попытаюсь понять ))


        1. Tyusha Автор
          04.10.2017 02:06

          1. Со скоростью света они двигаться не могут. Их скорость всегда будет чуточку меньше.

          2. Ничего экстраординарного с этими частицами происходить не будет, за исключением замедления физических процессов в них. В частности это проявится в увеличении их времени жизни.


          1. ZloyDooh
            04.10.2017 10:41

            Это я тоже на веру должен воспринять? )) То есть радиоактивные материалы можно стабилизировать, разогнав их до околосветовой скорости? Я верно понял идею, заслуживающую Нобелевской премии?
            Если серьезно, то троллить Вас забавно, хотя обычно я такими вещами не занимаюсь.
            А на веру принимать что-либо чревато тем, что тебя обманут )) Так что я поверю всему тому, что Вы написали, после предоставления Вами экспериментальных данных, подтвержденных и опубликованных в авторитетном издании ))
            Спасибо за внимание.


            1. photino
              04.10.2017 12:58

              То есть радиоактивные материалы можно стабилизировать, разогнав их до околосветовой скорости?

              Можно, но зачем?
              Это я тоже на веру должен воспринять? ))

              Да не должны, никто вас не заставляет. Не знаю как это видится со дна (как вы написали) именно вашего юридического образования, но люди с образованием не со дна, в том числе юридическим, обычно хотя бы прислушиваются к тем, кто в чем-то разбирается на порядки лучше, чем они сами.
              Так что я поверю всему тому, что Вы написали, после предоставления Вами экспериментальных данных, подтвержденных и опубликованных в авторитетном издании ))

              Ну вы типа барин, а остальные должны вам предоставлять что-то по вашему требованию? Хорош, вы такую тривиальщину спрашиваете, что она есть не то что в учебниках, а даже в научпопе. Ну или погуглите, это просто. Естественно, все это давно опубликовано в авторитетных журналах (правда как вы оцените авторитетность журналов по физике, совсем не ясно). Но нафиг кому надо искать ссылки для вас, если вы просто троллите (сами написали).


              1. ZloyDooh
                04.10.2017 13:42

                Скажу честно (и уже сказал) — троллю.
                Повторюсь, что причиной этого стал громкий заголовок («Нет ничего проще»).
                Скажу еще раз, что от физики очень далек, но любопытен до ужаса.
                Интерес к физике потерял, когда учитель физики не смогла объяснить на школьном уроке причину невозможности преодоления скорости света (невозможно, и всё).
                Моя попытка потроллить вызвана интересом к старому, заданному еще в школьные годы, вопросу. Если честно, то я жду объяснений, доступных моему пониманию, и стараюсь найти ошибки или то, что мне видится ошибками. Неспособность автора объяснить очевидные ему вещи простым языком говорит о том, что он не до конца понимает суть эффектов и явлений, либо недостаточно владеет языковым аппаратом.

                <Ну вы типа барин, а остальные должны вам предоставлять что-то по вашему требованию?>
                Никто мне ничего предоставлять не должен, но если не может объяснить, то зачем вообще пост писать нужно было?

                Гуглить, конечно, хорошо, но популяризаторы науки для того и существуют, чтобы язык ученых переводить на нормальный-бытовой. Статей на эту тему (научно-популярных) я не нашел. Вот и троллю.
                А автор теперь интерес потерял. (На мой взгляд, уклонилась от дискуссии по той же самой причине, что и мой школьный учитель много лет назад.)


                1. photino
                  04.10.2017 14:00

                  Неспособность автора объяснить очевидные ему вещи простым языком говорит о том, что он не до конца понимает суть эффектов и явлений, либо недостаточно владеет языковым аппаратом.

                  Это не совсем так, хотя и общепринятая точка зрения. Некоторые вещи просто невозможно объяснить на бытовом уровне, для этого нужно иметь некий общий понятийный аппарат. Поэтому часто в научпопе встречаются упрощения, которые вроде бы дают иллюзию понимания для человека не в теме, но на самом деле толком ничего не объясняют.
                  Это не пафос, ну вот просто не объяснить человеку, не знающему арифметику, как считается сдача в магазине. Можно, конечно, попробовать и потратить на это кучу времени, но проще сначала отправить его поучиться в школу.
                  Никто мне ничего предоставлять не должен, но если не может объяснить, то зачем вообще пост писать нужно было?

                  Думаю, что пост скорее для тех, кто хоть немного в теме. Ну не ориентироваться же всегда и везде абсолютно на всех.
                  Интерес к физике потерял, когда учитель физики не смогла объяснить на школьном уроке причину невозможности преодоления скорости света (невозможно, и всё).

                  Ну значит и интерес был не такой уж и настоящий (без обид). Самостоятельно ведь тоже можно много чего узнать и понять.
                  Что касается невозможности достижения скорости света массивным объектом, все упирается в то, что для этого нужна бесконечная энергия. Собственно, вот и все. А почему нужна бесконечная энергия — ну вот так природа устроена. А чтобы это действительно попробовать это понять, лучше все же потратить время, вспомнить школьную физику и попытаться хотя бы немного разобраться в той же СТО. К сожалению, просто только кошки родятся…
                  А автор теперь интерес потерял. (На мой взгляд, уклонилась от дискуссии по той же самой причине, что и мой школьный учитель много лет назад.)

                  Причина может быть не в невозможности или нежелании объяснить, а скорее в манере, в которой вы задавали вопросы.


                  1. ZloyDooh
                    04.10.2017 14:14

                    Из собственного опыта скажу, что через много лет после окончания школы мне случалось прочитать книги, после которых впору воскликнуть «Ну почему в школе так не объясняли!?».
                    Общие принципы мне известны (это о невозможности достижения скорости света массивным объектом и порядках энергии, необходимых для этого).
                    Увидев заголовок, я ожидал логичного и научного объяснения в тексте, но не увидел ни того, ни другого. А жаль.


                1. Druu
                  04.10.2017 14:31

                  > Интерес к физике потерял, когда учитель физики не смогла объяснить на школьном уроке причину невозможности преодоления скорости света (невозможно, и всё).

                  Возьмите листочек бумаги, поставьте в центре точку (это будет ноль) и проведите горизонтальную и вертикальную черту через нее, получите обычный график. Пусть горизонтальная линия — это пространственная координата х, а вертикальная — временная координата t. Тело, движущееся в этом пространстве с постоянной скоростью — некая прямая, наклон прямой — есть скорость тела, чем выше наклон — тем выше скорость. В данном случае вы можете сделать любой наклон, вплоть до прямой, параллельнок оси х — то есть бесконечная скорость.

                  Теперь возьмите и скукожьте лист вдоль оси х к центру, так, что профиль листа будет как у песочных часов — теперь как бы вы не проводили линию на листе, ее наклон не может превысить наклон полученных «песочных часов», т.к. тогда линии придется «выйти» за пределы листочка. А она этого сделать не может, так как на нем нарисована. Вот наше пространство — именно скукоженный таким образом листочек, наклон «песочных часов» — соответствует наклону прямой для тела, движущегося со скоростью света. Как вы не проводите линию — вы наклона большего не получите и скорости большей не получится. Не существует просто в пространстве прямых с бОльшим наклоном, их не выходит нарисовать.


                  1. ZloyDooh
                    04.10.2017 14:54

                    С трудом представляю связь между нашим пространством и невозможностью превзойти скорость света. Для того, чтобы выявить, что что-то движется со скоростью, большей скорости света, нужно это что-то сначала найти как минимум два раза, потом разделить расстояние между точками выявления на время, прошедшее между обнаружениями и сравнить со скоростью света. ))
                    Модель Ваша наглядна, но может быть очень далека от действительности.


                    1. Druu
                      04.10.2017 15:13

                      > С трудом представляю связь между нашим пространством и невозможностью превзойти скорость света.

                      Ну если вы в том или ином смысле «превзойдете», то вылетите за пределы «песочных часов».

                      > Модель Ваша наглядна, но может быть очень далека от действительности.

                      Может, конечно, модель и не должна быть близкой к действительности, наука изучением действительности не занимается. Модель должна быть непротиворечивой и давать валидные предсказания, на данный момент СТО — единственная такая модель. Точнее — фрагмент более крупной модели, и заменить этот фрагмент способа никакого нет (хотя нельзя скзаать, что никто не ищет — конечно, ищут :)).


                    1. itlen
                      04.10.2017 15:36

                      Совсем не далека. Световой конус


                      1. ZloyDooh
                        04.10.2017 16:43

                        Я не совсем правильно выразился. Не «далека от действительности», а «не описывает всех случаев для данной действительности».
                        Ведь физики-теоретики утверждают, что возможны перемещения со сверхсветовой скоростью и ищут этому доказательства. И чего только не придумают: и кротовые норы (червоточины), и искривление пространства-времени…
                        Терзают меня смутные сомнения, что источником энергии может являться объект, обладающий чудовищной гравитационной силой (сверхмассивная черная дыра, например). А энергию эту можно высвободить, достигнув некоей критической массы, при которой суммарная мощность объекта превысит энергию взаимодействия частиц, из которых объект состоит (что-то вроде атомного взрыва). Я так понимаю, что Вселенная и возникла в результате такого взрыва. Только не совсем понимаю, куда она расширяется.
                        И еще один момент.
                        Если исходить из того, что что-то находится за пределами светового конуса, то волновым способом измерения мы этого обнаружить не можем. Не так ли?


                      1. ZloyDooh
                        04.10.2017 16:45

                        И это не говорит о том, что что-то, не обнаружимое на данный момент при помощи волновых методов измерения, не может находится за пределами светового конуса.


                        1. Druu
                          04.10.2017 18:43

                          > И это не говорит о том, что что-то, не обнаружимое на данный момент при помощи волновых методов измерения, не может находится за пределами светового конуса.

                          «за пределами» нету пространства-времени, то есть даже самого понятия «за пределами» не существует. Световой конус — это, актуально, все, что есть.


                    1. Tyusha Автор
                      04.10.2017 18:01

                      Может поможет моё объяснение максимальности скорости света. Суть в том, что «скорость света» бесконечна, просто исторически, будучи расой не достигшей больших скоростей, мы неправильно выбрали единицы измерения.

                      Метры делённые на секунды — плохой вариант. Лучше переопределить скорость как V/sqrt{1-V2/c2).

                      Такая «скорость» разрешена до бесконечно, поэтому ни у кого не будет батхерта от того, что есть предел. Собственно говоря, это импульс.

                      Другое фантастическое построение. Давайте длину измерять в попугаях. Соответствие с нашими метрами такое

                      P [попугаев] = M [метров] * (1 — М [метров] / 38).

                      Тогда получится, что максимальная длина в мире не больше 38-ми попугаев. И у всех будет рвать крышу: как так, почему не может быть 39 попугаев.

                      Вот такая аналогия с предельностью скорости света. Возьмите вместо неё импульс (делённый на массу покоя) и спите спокойно.


                      1. Tyusha Автор
                        04.10.2017 18:14

                        Формула для попугаев выше неправильная, вот так надо:

                        P = M / (1 + M/38)


              1. ZloyDooh
                04.10.2017 13:44

                <То есть радиоактивные материалы можно стабилизировать, разогнав их до околосветовой скорости?

                Можно, но зачем?>

                Думаю, неплохое оружие бы получилось. Планетарного масштаба. Без шуток.


            1. Druu
              04.10.2017 14:25
              +1

              > То есть радиоактивные материалы можно стабилизировать, разогнав их до околосветовой скорости?

              Вы не поверите — но можно стабилизировать и хитрее, например, просто часто атом измерять. В пределе, если атом измерять бесконечно часто — он вообще никогда и не распадется ;)


              1. ZloyDooh
                04.10.2017 14:37

                ЛОЛ


                1. Druu
                  04.10.2017 15:13
                  +1

                  это не шутка -.-'


                  1. ZloyDooh
                    04.10.2017 16:17

                    Да я и не спорю, до достижения предела, конечно. А вот за пределами атома вообще не существует ))


  1. Tyusha Автор
    04.10.2017 11:14

    То есть радиоактивные материалы можно стабилизировать, разогнав их до околосветовой скорости?


    Да.


  1. ZloyDooh
    04.10.2017 11:55

    Примеры?


    1. Tyusha Автор
      04.10.2017 12:34

      Распад космических лучей, коллайдер.


      1. ZloyDooh
        04.10.2017 12:44

        А где в этих явлениях радиоактивные материалы?


        1. Tyusha Автор
          04.10.2017 12:55

          Знаете, мне беседа стала не интересна. Извините. Не хочу больше тратить на неё время.


          1. ZloyDooh
            04.10.2017 13:43

            Жду с нетерпением Ваших новых псевдо-научных статей.


  1. tundrawolf_kiba
    04.10.2017 13:42

    Я тут просто спросить. Я правильно понимаю, что если двигаться со скоростью света(насколько помню, это невозможно в рамках текущих знаний, но все же) — то расстояние в допустим 50 световых лет мы преодолеем мгновенно, но для внешнего наблюдателя пройдет 50 лет?


    1. photino
      04.10.2017 13:46

      Что-то в этом роде. Скорости света не достигнете, но сделать время перелета по часам такого космонавта меньше любого наперед заданного положительного времени теоретически можно. А для внешнего наблюдателя да, пройдет 50 лет.


      1. ZloyDooh
        04.10.2017 14:02

        Так о том я и говорю, что сократится не расстояние (L), а, (возможно), время, которое пройдет для путешественника относительно периода времени, истекшего для наблюдателя. Но все это в настоящее время практически не проверено. Или я ошибаюсь?


        1. photino
          04.10.2017 14:14

          Расстояние в его системе отсчета по его линейке тоже сократится (если система инерциальная, такое измерения расстояния можно спокойно проделать).
          Проверено совершенно разными способами. Самое простое — вышеупомянутый эксперимент Хафеле — Китинга (см. википедию, например). Или тот же GPS.


          1. ZloyDooh
            04.10.2017 14:36

            А как же однородность времени в инерциальных системах?


            1. Tyusha Автор
              04.10.2017 17:44
              +1

              С однородностью времени в ИСО всё в порядке. Однородность времени ведёт по теореме Нётер к сохранению энергии. Отклонений не обнаружено